80 neuro resp gastro haem questions

66
www.AceMedicine.com 80 Neurology, Respiratory, Gastorenterology, Haematology SBAs, MCQs & EMQs Neurology SBA A 32-year-old woman presents with intermittent headaches lasting around 24 hours, associated with nausea and sensitivity to movement, light and sound. The frequency of the headaches is four or five per month. What is the best prophylactic treatment to give her to try and prevent the headaches? A. Sumatriptan B. Paracetamol C. Verapamil D. Ibuprofen E. Propranolol E. Propranolol Management of migraine involves treatment of the acute episode and prophylactic treatment in patients with regular headaches (usually greater than 3 per month). Acute treatment is with simple analgesia (and antiemetic if necessary) initially, but some patients may benefit from using a triptan. A number of different drugs can be used for prophylactic treatment including propranolol, amitriptyline, sodium valproate and topiramate. Neurology MCQ A 22-year-old woman presents with intermittent headaches that occur infrequently, around once every month and last for around 24 hours every time. They are usually unilateral, throbbing in nature with associated nausea, photophobia and phonophobia. Which of the following drugs might be used in treating an acute attack of this type of headache? A. Sumatriptan B. Amitriptyline C. Aspirin D. Propranolol E. Pizotifen A and C might be used in an acute migraine attack. Initially simple analgesia (+/- an antiemetic) are used for migraine, and aspirin is one of the most effective of the simple analgesics for migraine. If these do not help then the triptans may be used these include sumatriptan, naratriptan and rizatriptan. The other drugs mentioned here are only used as preventative treatments for migraine. Other drugs apart from propranolol, pizotifen and amitriptyline used for migraine prophylaxis include sodium valproate and topiramate.

Upload: salemsa

Post on 22-Oct-2015

47 views

Category:

Documents


2 download

TRANSCRIPT

Page 1: 80 Neuro Resp Gastro Haem Questions

www.AceMedicine.com

80 Neurology, Respiratory, Gastorenterology, Haematology SBAs, MCQs & EMQs

Neurology SBA A 32-year-old woman presents with intermittent headaches lasting around 24 hours, associated with nausea and sensitivity to movement, light and sound. The frequency of the headaches is four or five per month. What is the best prophylactic treatment to give her to try and prevent the headaches?

A. Sumatriptan B. Paracetamol C. Verapamil D. Ibuprofen E. Propranolol

E. Propranolol Management of migraine involves treatment of the acute episode and prophylactic treatment in patients with regular headaches (usually greater than 3 per month). Acute treatment is with simple analgesia (and antiemetic if necessary) initially, but some patients may benefit from using a triptan. A number of different drugs can be used for prophylactic treatment including propranolol, amitriptyline, sodium valproate and topiramate. Neurology MCQ A 22-year-old woman presents with intermittent headaches that occur infrequently, around once every month and last for around 24 hours every time. They are usually unilateral, throbbing in nature with associated nausea, photophobia and phonophobia. Which of the following drugs might be used in treating an acute attack of this type of headache?

A. Sumatriptan B. Amitriptyline C. Aspirin D. Propranolol E. Pizotifen

A and C might be used in an acute migraine attack. Initially simple analgesia (+/- an antiemetic) are used for migraine, and aspirin is one of the most effective of the simple analgesics for migraine. If these do not help then the triptans may be used – these include sumatriptan, naratriptan and rizatriptan. The other drugs mentioned here are only used as preventative treatments for migraine. Other drugs apart from propranolol, pizotifen and amitriptyline used for migraine prophylaxis include sodium valproate and topiramate.

Page 2: 80 Neuro Resp Gastro Haem Questions

www.AceMedicine.com

Neurology SBA A 25-year-old woman has an episode of altered consciousness lasting a few minutes. She had no memory of the event, but just beforehand she developed a sense of déjà vu and had a rising feeling in her stomach.

A. What is the most likely cause? B. Complex partial seizure C. Cardiac arrhythmia D. Postural hypotension E. Vasovagal syncope F. Transient global amnesia

A. Complex partial seizure She has features which are characteristic ‘aura’ that occur before complex partial seizures arising from the temporal lobe i.e. déjà vu and a rising epigastric sensation. The other causes may lead to altered or loss of consciousness, but not the characteristic aura. Syncope, either due to vasovagal episodes or cardiac causes often have presyncopal symptoms including lightheadedness prior to losing consciousness. Neurology MCQ A 32 year-old man is known to have complex partial seizures consisting of brief episodes of altered consciousness associated with olfactory hallucinations, episodes of déjà vu and a rising epigastric sensation. Which of the following drugs may be helpful in controlling his seizures?

A. Carbamazepine B. Sodium valproate C. Propranolol D. Amitriptyline E. Lamotrigine

A, B and E are commonly used antiepileptic medications which may be used in complex partial seizures. Carbmazepine is commonly used as a first-line medication in patients with partial seizures but other drugs including sodium valproate and lamotrigine can be used as well. C and D are not antiepileptic medications. Propranolol and amitriptyline are both commonly used for migraine prophylaxis.

Page 3: 80 Neuro Resp Gastro Haem Questions

www.AceMedicine.com

Neurology SBA A 35-year-old woman presents to neurology clinic after developing odd movements of the limbs and face over the last nine months. Her father had similar problems and died in his 50’s. On examination she had grimacing of the face and involuntary, irregular movements of the arms and legs. What is the most likely diagnosis?

A. Wilson’s disease B. Huntington’s disease C. Parkinson’s disease D. Sydenham’s chorea E. Multiple system atrophy

B. Huntington’s disease This woman has a hyperkinetic movement disorder consistent with chorea. With her father suffering from the same illness, it raises the possibility of an autosomal dominant disorder with the most likely cause being Huntington’s disease. In Sydenham’s chorea a family history would not be expected and the other disorders are predominantly hypokinetic movement disorders rather than hyperkinetic (although tremor may be seen in each). Neurology MCQ A 72-year-old man has Parkinson’s disease with an asymmetrical akinetic-rigid syndrome and rest tremor. Which of the following drugs might you use as therapy for him?

A. Levodopa B. Tetrabenazine C. Propranolol D. Pramipexole E. Ropinirole

A, D and E. Levodopa and the dopamine agonists pramipexole and ropinirole are commonly used as first line therapy for Parkinson’s disease. In younger people the dopamine agonists are usually used first-line whereas in older patients levodopa is used. Tetrabenazine can be used in patients with chorea but is unhelpful for akinetic-rigid syndromes whilst propranolol is commonly used in patients with essential tremor and similarly is not useful for akinetic-rigid syndromes.

Page 4: 80 Neuro Resp Gastro Haem Questions

www.AceMedicine.com

Neurology SBA A 25-year-old man had been hit on the head by a cricket ball and then about one hour later had lost consciousness. On a CT head scan in A+E he had a haemorrhage shaped like a convex lens. What is the most likely cause?

A. Subdural haematoma B. Subarachnoid haemorrhage C. Extradural haematoma D. Brainstem haemorrhage E. Intracerebral haemorrhage

C. Extradural haematoma He has a clinical history often seen in patients with extradural haematomas who have a lucid interval following the head injury before losing consciousness. The CT scan shows the features typical of an extradural haematomas where the bleed is ‘lentiform’ i.e. shaped like a lens. This differentiates it from other types of bleeds e.g. subdural haematomas which are may be shaped like a ‘crescent’, subarachnoid haemorrhage where the blood is seen in the subarachnoid space and intracerebral haemorrhages where the blood is seen within the brain. Neurology SBA A 32-year-old woman sustains a head injury in a road traffic accident. On examination in A+E she is only making a few, unclear and incomprehensible sounds and she opens her eyes only to a painful stimulus. She will move her arm in order to localise to a painful stimulus. What is her Glasgow Coma Scale score?

A. 7 B. 8 C. 9 D. 10 E. 11

C. 9. She scores 2 for eye response, 2 for verbal response and 5 for her motor response. The maximum GCS is 15 and is scored as best eye response out of 4, best verbal response out of 5 and best motor response out of 6. Eye response is scored as 1 no eye opening, 2 eye opening in response to pain, 3 eye opening to command, 4 eyes open spontaneously. Verbal response is scored as 1 no verbal response, 2 incomprehensible sounds, 3 inappropriate words, 4 confusion, 5 normal speech. Motor response is scored as 1 no motor response, 2 extension to pain, 3 abnormal flexion to pain, 4 normal flexion to pain, 5 localization to pain, 6 obeys commands.

Page 5: 80 Neuro Resp Gastro Haem Questions

www.AceMedicine.com

Neurology SBA A 34-year old woman presents to A+E with a two day history of increasing headache and confusion. She has a temperature of 38.2 degrees. Whilst in the emergency department she has a tonic-clonic seizure. MRI shows increased signal in the temporal lobes on T2 imaging. What is the most likely cause?

A. Alcohol intoxication B. Severe pneumonia C. Viral encephalitis D. Subarachnoid haemorrhage E. Hepatic encephalopathy

B. Viral encephalitis She has acute confusion with a headache, seizure and a fever. Combined with the imaging findings this is characteristic for a viral encephalitis. The other disorders may cause acute confusion but not the other features in combination. Brain imaging would be expected to be normal in the other cases apart from subarachnoid haemorrhage where there may be evidence of a bleed on the scan. A temperature and confusion may be seen in severe pneumonia but not a seizure. Alcohol intoxication may lead to seizures, particularly with withdrawal but is unlikely to cause a fever of 38.2. Neurology MCQ A 72-year-old woman presents to A+E acutely confused. Which of the following are possible causes?

A. Urinary tract infection B. Seizure C. Pneumococcal meningitis D. Renal failure E. Hypoglycaemia

All of the above causes may cause an acute confusional state. Causes can be thought of: 1) Metabolic and electrolyte abnormalities including hyponatraemia, hypercalcaemia, hepatic encephalopathy, uraemia, hypoglycaemia, 2) Infections such as UTIs or pneumonia, 3) Drugs and toxins e.g. alcohol or recreational drugs, 4) Neurological causes such as seizures, stroke or encephalitis, 5) Hypoxia.

Page 6: 80 Neuro Resp Gastro Haem Questions

www.AceMedicine.com

Neurology SBA A 53-year-old woman has a one-week history of sudden onset of weakness of the left side of her face. On examination she has a left-sided facial droop with difficulty on the left side in raising her eyebrow, closing her eye and blowing out her cheek. What is the most likely diagnosis?

A. Myasthenia gravis B. Lacunar infarct C. Multiple sclerosis D. Bell’s palsy E. Middle cerebral artery infarct

D. Bell’s palsy This patient has weakness affecting all muscle groups on the left side of her face including the forehead suggestive of a lower motor neurone lesion (a 7th nerve palsy). The most common cause for this would be a Bell’s palsy. The other options can all cause facial weakness – hemispheric infarcts will cause an upper motor neurone facial weakness with forehead sparing. Myasthenia gravis usually causes bilateral facial weakness and there may be other features of myasthenia including ptosis and ophthalmoplegia. Facial weakness is rare in multiple sclerosis and is usually an upper motor neurone lesion. Neurology MCQ A 32-year-old develops right-sided lower motor neurone facial weakness. Which of the following are possible causes of this?

A. Lacunar infarct B. Parotid gland tumours C. Guillain-Barre syndrome D. Bell’s palsy E. Middle cerebral artery infarct

B, C and D may all cause a lower motor neurone facial weakness whilst A and E cause upper motor neurone facial weakness. Facial weakness may be caused by a lesion in the upper motor neurone (from the motor cortex to the 7th nerve nucleus in the brainstem e.g. strokes affecting the cerebral hemispheres), the lower motor neurone (i.e. the 7th nerve itself, from the 7th nerve nucleus to the neuromuscular junction e.g. Bell’s palsy, Guillain-Barre syndrome, Lyme disease, sarcoidosis, parotid gland tumours), the neuromuscular junction (e.g. myasthenia gravis) and the muscle (e.g. muscular dystrophies such as myotonic dystrophy).

Page 7: 80 Neuro Resp Gastro Haem Questions

www.AceMedicine.com

Neurology SBA A 51-year-old man with diabetes develops features of an autonomic neuropathy. Which of the following symptoms does not occur in an autonomic neuropathy?

A. Erectile dysfunction B. Urinary urgency C. Postural hypotension D. Stridor E. Constipation

D. Stridor All the other features may be seen in patients with autonomic neuropathy. Symptoms may include abnormal sweating as well as cardiovascular problems such as postural hypotension, gastrointestinal problems such as diarrhoea or constipation, urinary dysfunction such as urgency or frequency, and erectile dysfunction. It occurs in a number of different diseases including diabetes, amyloidosis and Guillain-Barre syndrome. Neurology MCQ Which of the following disorders may cause an autonomic neuropathy?

A. Guillain-Barre syndrome B. Diabetes C. Charcot-Marie-Tooth disease D. Amyloidosis E. Sarcoidosis

A, B and D may cause an autonomic neuropathy. These are less common than other neuropathies but can occur in a number of different conditions, particularly diabetes, amyloidosis and Guillain-Barre syndrome. Patients with autonomic neuropathies may have a wide variety of symptoms including cardiovascular problems e.g. postural hypotension, gastrointestinal problems e.g. diarrhoea or constipation, erectile dysfunction, abnormal sweating and urinary dysfunction.

Page 8: 80 Neuro Resp Gastro Haem Questions

www.AceMedicine.com

Neurology SBA A 67-year-old woman had a 3 month history of weakness of his left leg and a 1 month history of weakness in both arms. He felt things were getting progressively worse. On examination he had wasting and fasciculations in all four arms. He was weak in all four limbs with brisk reflexes throughout with upgoing plantars. He had a normal sensory examination. What is the most likely diagnosis?

A. Polymyositis B. Parkinson’s disease C. Multiple sclerosis D. Motor neurone disease E. Myasthenia gravis

D. Motor neurone disease He has progressive weakness with a mix of upper and lower motor neurone signs on examination in the presence of a normal sensory exam. This is consistent with motor neurone disease. Parkinson’s disease does not cause weakness. Multiple sclerosis is a central nervous system disorder and therefore associated with upper motor neurone signs. Myasthenia gravis is a neuromuscular junction disorder and polymyositis is a muscle disorder, neither of which would be associated with pathologically brisk reflexes (an upper motor neurone sign). Neurology MCQ A 67-year-old man has motor neurone disease. Which of the following features would be seen in a pseudobulbar palsy?

A. Tongue wasting and fasciculations B. Dysphagia C. Brisk jaw jerk D. Spastic tongue E. Absent gag reflex

B, C and D are seen in a pseudobulbar palsy, an upper motor neurone disorder whilst A and E (as well as B) may be seen in a bulbar palsy, a lower motor neurone disorder. In a pseudobulbar palsy there is a spastic dysarthria, dysphagia, brisk jaw jerk and a spastic tongue. In a bulbar palsy, there is a nasal dysarthria, dysphagia, absent gag reflex and a wasted tongue with fasciculations.

Page 9: 80 Neuro Resp Gastro Haem Questions

www.AceMedicine.com

Neurology SBA A 53-year-old woman presents to the neurology clinic with a six month history of difficulty getting up from a chair. More recently she has noticed difficulty lifting things. On examination she has a rash over the dorsum of her hands and both eyelids. What is the most likely diagnosis?

A. Motor neurone disease B. Vitamin B12 deficiency C. Dermatomyositis D. Myotonic dystrophy E. Myasthenia gravis

C. Dermatomyositis. This patient gives a history of proximal limb weakness, usually associated with muscle disorders. With the associated rash this would be most likely to be dermatomyositis. The description is of Gottron’s papules, a rash over the knuckles, and the heliotrope rash over the eyelids, both characteristic of dermatomyositis. A rash would not be expected in the other disorders. Neurology MCQ A 45-year-old man develops proximal limb weakness. Which of the following disorders may cause a myopathy?

A. Cushing’s disease B. Hypothyroidism C. Osteoporosis D. Thyrotoxicosis E. Turner’s syndrome

A, B and D can all cause a myopathy i.e. a disorder of muscles that commonly causes proximal symmetrical limb weakness and/or wasting. Endocrinopathies are common causes of myopathies and other endocrine myopathies include hyperparathyroidism, Addison’s disease and acromegaly. The other common causes of myopathies include inflammatory disorders such as polymyositis and dermatomyositis, and drugs such as steroids and alcohol.

Page 10: 80 Neuro Resp Gastro Haem Questions

www.AceMedicine.com

Neurology SBA A 45-year-old man presents with a one year history of difficulty walking as well as pain and sensory disturbance in the upper limbs. On examination he has a spastic paraparesis and in the upper limb decreased sensation to pain in a ‘cape-like’ distribution i.e. over the shoulders, the lateral aspects of both arms and both thumbs and index fingers with intact proprioception and vibration. What is the most likely cause of his problems?

A. Multiple sclerosis B. Syringomyelia C. Vitamin B12 deficiency D. Anterior spinal artery infarct E. Tuberculosis

B. Syringomyelia The patient has a spastic paraparesis which can be seen in each of the conditions but has a dissociated sensory loss in a ‘cape-like’ distribution which is characteristic of a syrinx in the cervical region. Vitamin B12 deficiency may cause subacute combined degeneration of the cord with dorsal column involvement but sparing of the spinothalamic tract. Anterior spinal artery infarcts may only affect the anterior part of the cord i.e. affect the spinothalamic tract but not the dorsal columns. MS and TB may variably affect the cord but may affect both the spinothalamic tracts and dorsal columns. Neurology MCQ A 52-year-old woman presents with difficulty walking for six months and has features characteristic of a spastic paraparesis on examination. Which of the following disorders might cause this?

A. Multiple sclerosis B. Syringomyelia C. Guillain-Barre syndrome D. Transverse myelitis E. Charcot-Marie-Tooth disease

A, B and D may cause a spastic paraparesis i.e. they are upper motor neurone disorders that can affect the corticospinal tracts within the spinal cord. C and E are lower motor neurone disorders and affect the peripheral nerve causing the features of a peripheral neuropathy. Guillain-Barre syndrome is an acute onset lower motor neurone disorder causing ascending weakness whilst Charcot-Marie-Tooth disease is a genetic peripheral neuropathy causing distal symmetrical weakness and/or distal sensory disturbance.

Page 11: 80 Neuro Resp Gastro Haem Questions

www.AceMedicine.com

Neurology EMQ 1

A. Brown-Sequard syndrome B. Peripheral neuropathy C. Myopathy D. Brainstem lesion E. Spinal cord compression F. Anterior spinal cord lesion G. Neuromuscular junction disorder H. Plexopathy I. Posterior spinal cord lesion J. Motor cortex lesion

Choose which of the above causes is the most appropriate location for each of the descriptions of a neurological deficit. Q1. A 25-year-old man returns from holiday to Spain and develops sudden weakness of the legs followed a few days later by weakness in the arms. On examination he has weakness in all four limbs and his reflexes are absent. A1. Answer B. This man most likely has Guillain-Barre syndrome (GBS) i.e. a peripheral neuropathy. His reflexes are absent - a lower motor neurone lesion. Acute ascending weakness in a lower motor neurone pattern is most likely caused by GBS. Q2. A 65-year-old woman develops weakness in both legs. She has weakness mostly of the flexor muscles in a symmetrical pattern with brisk reflexes. She also has vibration and proprioception impairment in the legs but intact pain and temperature sensation. A2. Answer I. This woman has a posterior spinal cord lesion characterized by dorsal column impairment with intact spinothalamic tracts. Q3. A 50-year-old woman has increasing difficulty rising from a chair over a two month period. On examination she has symmetrical proximal weakness of the legs and also the arms with some pain in the same muscles. A3. Answer C. This woman has a myopathy characterized by symmetrical proximal weakness. Q4. A 71-year-old man with known prostate cancer presents with increasing difficulty walking over the last couple of weeks. He has weakness of the flexor muscles in the legs and brisk reflexes. All sensory modalities are impaired in the legs with light touch and pinprick sensation impaired onto the abdomen up to the umbilicus.

Page 12: 80 Neuro Resp Gastro Haem Questions

www.AceMedicine.com

A4. Answer E. This gentleman has a spinal cord lesion affecting the whole cord (corticospinal tracts, dorsal columns and spinothalamic tracts). He has a sensory level suggesting a lesion affecting the spinal cord about T10. Q5. A 50-year-old woman presents with difficulty keeping her head up. She has neck flexion weakness on examination as well as proximal weakness in the arms which is fatigable. A5. Answer G. This lady has myasthenia gravis, a neuromuscular junction disorder, with characteristic fatigability. Neurology EMQ 2

A. Broca’s aphasia B. Pseudobulbar dysarthria C. Cerebellar dysarthria D. Wernicke’s aphasia E. Conduction aphasia F. Cerebellar dysarthria G. Bulbar dysarthria H. Hypokinetic (Parkinsonian) dysarthria I. Transcortical sensory aphasia

Choose which of the above causes is the most appropriate speech abnormality for each of the descriptions below. Q1. A 35-year-old woman has slurred speech associated with unsteadiness in walking. A1. Answer C. This lady has slurring of speech in association with an ataxic gait, both features of a cerebellar disorder. Q2. A 61-year-old man has difficulty in producing speech. His speech contains incorrect words that are similar in sound to the correct word. His speech is also missing function words such as ‘and’, ‘the’ and ‘of’. Comprehension of speech is normal. A2. Answer A. This man has a nonfluent or expressive aphasia with speech errors and agrammatism characteristic of Broca’s aphasia in contrast to the fluent or receptive aphasia seen in Wernicke’s aphasia. Q3. A 42-year-old man has nasal speech associated with a wasted tongue. A3. Answer G. This man has features of a bulbar palsy, a lower motor neurone syndrome, contrasting with the upper motor neurone features seen in a pseudobulbar palsy.

Page 13: 80 Neuro Resp Gastro Haem Questions

www.AceMedicine.com

Q4. A 75-year-old man has difficulty with repeating words but relatively normal speech production apart from the occasional speech error. Comprehension of speech is normal. A4. Answer E. This man has relatively intact speech production and comprehension but impaired repetition – this is consistent with conduction aphasia (and distinct to either Broca’s aphasia where there is speech production impairment and Wernicke’s aphasia where there is speech comprehension impairment). Q5. A 69-year-old man has very soft, quiet speech that trails off towards the end of a sentence. The content of his speech is normal as is comprehension. A5. Answer H. This man has the type of speech disorder seen in Parkinsonian disorders. Neurology EMQ 3

A. Idiopathic Parkinson’s Disease B. Essential tremor C. Huntington’s disease D. Hemiballismus E. Drug-induced parkinsonism F. Progressive supranuclear palsy G. Wilson’s disease H. Thyrotoxicosis I. Multiple system atrophy J. Sydenham’s chorea K. Creutzfeldt-Jakob disease L. Physiological tremor

Choose which of the above causes is the most appropriate movement disorder for each of the descriptions below. Q1. A 55-year-old man has facial grimacing and writhing movements of the arms. His father also had the same problem. A1. Answer C. This man has a hyperkinetic movement disorder which sounds like chorea. With a family history of the same problem the most likely cause would be Huntington’s disease. Q2. A 44-year-old woman has a tremor in both arms which is most prominent when her hands are outstretched but is not present when she is resting. She has noticed that it improves whenever she has a glass of wine. The rest of the neurological examination is normal.

Page 14: 80 Neuro Resp Gastro Haem Questions

www.AceMedicine.com

A2. Answer B. This woman has a symmetrical postural tremor which improves with alcohol. This is most likely to be an essential tremor. There are no other neurological abnormalities on examination to suggest a parkinsonian disorder Q3. A 65-year-old man has had rapidly progressive memory problems over the last few months and his arms have started intermittently jerking. A3. Answer K. This man has myoclonus of the arms. This occurs in a number of conditions but in association with a rapidly progressive dementia is likely to be caused by CJD. Q4. A 78-year-old woman has a tremor in her right arm which is most prominent at rest. She has also had increased difficulty using the right arm as it has become stiff over the last six months. A4. Answer A. An asymmetrical rest tremor is most characteristic of idiopathic Parkinson’s disease. This woman also described rigidity, one of the other features of PD along with bradykinesia. Q5. A 66-year-old man has difficulty walking with a number of falls backwards. Examination reveals relatively symmetrical rigidity and difficulty looking downwards. A5. Answer F. Progressive supranuclear palsy is one of the “Parkinson-plus” conditions with relatively symmetrical parkinsonism (compared to the asymmetry seen in idiopathic PD) and a supranuclear gaze palsy (an eye movement disorder). Neurology EMQ 4

A. Juvenile myoclonic epilepsy B. Hyponatraemia C. Hypoxia D. Venous sinus thrombosis E. Encephalitis F. Hypoglycaemia G. Hypocalcaemia H. Temporal lobe epilepsy I. Alcohol withdrawal seizures J. Glioma K. Head injury L. Idiopathic generalized epilepsy

Choose which of the above causes is the most appropriate cause of seizures for each of the descriptions below. Q1. A 42-year-old man presents with a few episodes of altered consciousness. He says that he gets an odd smell before the episodes and occasionally a rising feeling in his stomach.

Page 15: 80 Neuro Resp Gastro Haem Questions

www.AceMedicine.com

A1. Answer H. Temporal lobe epilepsy can be associated with a number of symptoms including déjà vu, jamais vu, a rising feel in the stomach and olfactory hallucinations. Q2. A 65-year-old man presents with confusion and a tremor and appears to be describing visual hallucinations. He then has a tonic-clonic seizure. A2. Answer I. This gentleman has features of an alcohol withdrawal syndrome. Q3. A 22-year-old man presents with three generalized tonic-clonic seizures over the last year. He mentions that since the age of 16 he will occasionally get jerking movements of the arms early in the morning. A3. Answer A. This man has myoclonus (jerking movements) of the arms, which started in his teens, in association with seizures – the most likely diagnosis is juvenile myoclonic epilepsy. Q4. A 65-year-old man presents with a tonic-clonic seizure. He had recently been started on a drug for hypertension. A4. Answer B. Hyponatraemia can cause seizures. This gentleman is likely to have been on a thiazide diuretic leading to SIADH and thus hyponatraemia. Q5. A 26-year-old woman presents with a headache and confusion and on examination was found to have papilloedema. She has a tonic-clonic seizure whilst still in A+E. A5. Answer D. The presence of headache, seizures and raised intracranial pressure (the patient has papilloedema) is seen in venous sinus thrombosis. This is more likely than encephalitis which may also cause headache and seizures. Neurology EMQ 5

A. Horner’s syndrome B. Third nerve palsy C. Adie’s pupil D. Opiate overdose E. Argyll Robertson pupil F. Marcus Gunn pupil G. Pontine haemorrhage H. Tricyclic antidepressant overdose

Choose which of the above causes is the most appropriate pupillary abnormality for each of the descriptions below.

Page 16: 80 Neuro Resp Gastro Haem Questions

www.AceMedicine.com

Q1. A 65-year-old man has a small left pupil and the eyelid on that side is drooping. He was also noted to have wasting of the small muscles of the hand on the left. A1. Answer A. This man has a Horner’s syndrome. Ptosis and pupil abnormalities are associated with Horner’s syndrome (small pupil) and third nerve palsy (large pupil). Q2. A 30-year-old woman has a normal pupillary size on observation but on shining the light into each eye in turn, the left pupil dilates rather than constricts. A2. Answer F. This woman has a relative afferent pupillary defect or a Marcus Gunn pupil. Unlike most other pupillary abnormalities the pupils are normal on observation. However, the ‘swinging flashlight’ test reveals a paradoxical dilation of the pupil on shining the light into the eye with the abnormal optic nerve. Q3. A 33-year-old woman is admitted to A+E with decreased consciousness. She is noted to have bilaterally dilated pupils. A3. Answer H. Tricyclic antidepressant overdose is associated with dilated pupils compared to opiates which cause constricted pupils. Q4. A 69-year-old man is admitted to A+E with a severe headache followed by a decrease in conscious level. He was noted to have bilaterally constricted pupils. A4. Answer G. Decreased conscious level with bilaterally constricted pupils can be due to a number of causes including opiate overdose and organophosphate poisoning but in a 69-year-old man with a preceding severe headache the most likely cause is a pontine haemorrhage. Q5. A 45-year-old woman has a dilated left pupil. Her neurological exam is otherwise normal apart from decreased reflexes throughout. A5. Answer C. A unilateral dilated pupil without a ptosis or eye movement disorder is most commonly due to an Adie’s pupil. Loss of deep tendon reflexes is often seen in this condition – Holmes-Adie syndrome.

Page 17: 80 Neuro Resp Gastro Haem Questions

www.AceMedicine.com

Neurology EMQ 6

A. Central scotoma B. Left homonymous hemianopia C. Right inferior homonymous quadrantanopia D. Left inferior homonymous quadrantanopia E. Bitemporal hemianopia F. Right superior homonymous quadrantanopia G. Left superior homonymous quadrantanopia H. Right homonymous hemianopia I. Tunnel vision

Choose which of the above causes is the most appropriate visual field defect for each of the descriptions below. Q1. A 72-year-old woman is found to have a right posterior cerebral artery infarct. A1. Answer B. Posterior cerebral artery infarcts are usually associated with homonymous hemianopias. Beyond the optic chiasm the visual hemifield is represented on the opposite side of the brain i.e. left hemifield in the right hemisphere. Q2. A 25-year-old woman develops a painful visual disturbance in her left eye. She had previously had an episode of ataxia a year earlier lasting a few months. A2. Answer A. This woman is likely to have multiple sclerosis (two episodes of focal neurological deficits separated in time and space). The latest deficit is likely to be an optic neuritis which may cause a central scotoma. Q3. A 45-year-old man develops enlarged hands and feet as well as coarsened facial features. A3. Answer E. This man has acromegaly due to a pituitary tumour. Pituitary tumours may press on the optic chiasm causing a bitemporal hemianopia. Q4. A 66-year-old man was found to have had a left parietal stroke. A4. Answer C. Beyond the optic chiasm the visual hemifield is represented on the opposite side of the brain i.e. right hemifield in the left hemisphere. The optic radiations pass through the temporal lobe (lower) and the parietal lobe (upper) and represent the opposite part of the field i.e. upper radiation represents the inferior field and vice versa. Q5. A 65-year-old man has impaired night vision and is found to have retinitis pigmentosa

Page 18: 80 Neuro Resp Gastro Haem Questions

www.AceMedicine.com

A5. Answer I. Retinitis pigmentosa causes impaired peripheral vision which may lead to tunnel vision. Neurology EMQ 7

A. Benign paroxysmal positional vertigo B. Vasovagal syncope C. Vestibular migraine D. Vertebrobasilar ischaemia E. Labyrinthitis F. Meniere’s disease G. Postural hypotension H. Cardiac arrhythmia

Choose which of the above causes is the most appropriate cause of dizziness for each of the descriptions below. Q1. A 68-year-old man presents with recurrent episodes of dizziness lasting a few hours each time and associated with hearing loss and tinnitus in his left ear. A1. Answer F. In Meniere’s disease there is vertigo associated with tinnitus and hearing loss. Q2. A 45-year-old presents with a feeling of the world spinning for the last few days. She has also vomited a few times. A2. Answer E. A ‘labyrinthitis’ or vestibular neuronitis causes vertigo lasting days, compared to BBPV which causes vertigo lasting seconds, and vertebrobasilar ischaemia lasting minutes and Meniere’s disease lasting hours. Q3. A 50-year-old woman has a feeling of dizziness first thing in the morning and on getting up from a chair. A3. Answer G. The most likely cause is postural hypotension. Q4. A 65-year-old woman has brief episodes of a feeling of the whole world spinning, particularly when turning in bed. A4. Answer A. BPPV is characterized by episodes of vertigo lasting seconds precipitated by position e.g. turning head to one side or turning in bed. Q5. A 32-year-old woman has recurrent episodes of a feeling of the whole world spinning lasting about half an hour. Occasionally these are associated with a right-sided headache.

Page 19: 80 Neuro Resp Gastro Haem Questions

www.AceMedicine.com

A5. Answer C. Recurrent episodes of vertigo associated with headache in a young person are most likely to be due to migraine. Neurology EMQ 8

A. Multiple sclerosis B. Dermatomyositis C. Common peroneal nerve palsy D. L5 disc prolapse E. Diabetic amyotrophy F. Myasthenia gravis G. Becker’s muscular dystrophy H. Charcot-Marie-Tooth disease I. Chronic inflammatory demyelinating polyneuropathy J. Vitamin B12 deficiency K. Brown-Sequard syndrome L. Anterior spinal artery infarction

Choose which of the above causes is the most appropriate cause of leg weakness for each of the descriptions below. Q1. A 32-year-old man develops a left foot drop after having had a cast on his leg following a fracture. A1. Answer C. A common peroneal palsy may develop following pressure on the nerve by a plaster cast. Q2. A 41-year-old woman develops difficulty in rising from a chair and has proximal weakness in her legs. She is also noted to have red areas over the backs of her fingers. A2. Answer B. This woman has a proximal myopathy with a rash on the back of her hands consistent with Gottron’s papules, features of dermatomyositis. Q3. A 32-year-old woman has symmetrical weakness In both legs particularly of the flexor muscles. She has brisk reflexes and upgoing plantars. A year earlier she had an episode of painful loss of vision lasting a month. A3. Answer A. This woman has a spastic paraparesis and sounds like she previously had an optic neuritis. The most likely diagnosis is multiple sclerosis (two episodes separated in time and space).

Page 20: 80 Neuro Resp Gastro Haem Questions

www.AceMedicine.com

Q4. A 33-year-old man has distal symmetrical wasting and weakness of both legs with bilateral pes cavus. A4. Answer H. This man has features of a peripheral neuropathy and in combination with pes cavus this is likely to be a hereditary neuropathy such as Charcot-Marie-Tooth disease. Q5. A 46-year-old man presents with weakness of his left leg. On examination of his sensation he has impaired proprioception and vibration in the left leg and impaired pain sensation in the right leg. A5. Answer K. This man has the classic features of a Brown-Sequard syndrome with dissociated sensory loss (dorsal column loss ipsilateral to the lesion and spinothalamic tract loss contralateral to the lesion).

Page 21: 80 Neuro Resp Gastro Haem Questions

www.AceMedicine.com

Respiratory MCQ A 44 year old man attends A & E following an acute episode of shortness of breath. A chest x-ray is performed.

Which of the following conditions predispose to this condition?

A. Pulmonary fibrosis B. Asthma C. COPD D. Kartagener’s syndrome E. Marfan’s Syndrome

T,T,T,F,T Pneumothorax is identified on chest x-ray by the presence of a lung edge peripheral to which is an area of lucency, which does not contain lung markings. The following are all predisposing factors: Congenital pulmonary blebs Syndromes, such as Marfan’s and homocysteinuria Chronic chest diseases, including asthma, cystic fibrosis, COPD, and sarcoidosis. Atypical infections ,such as PCP pneumonia Pneumothoraces can also occur following trauma or iatrogenic injury.

Page 22: 80 Neuro Resp Gastro Haem Questions

www.AceMedicine.com

Treatment depends on the size of the Pneumothorax, ranging from conservative treatment to immediate decompression with a pleural drain. Respiratory SBA A 59 year old lady with Grade 3 congestive cardiac failure has become increasingly short of breath despite excellent control of her cardiac function and no deterioration in her ejection fraction on echocardiography. A chest radiograph is reported to demonstrate bi-basal lung fibrosis which was not present 2 years earlier. Which of her cardiac medications is the most likely cause of the fibrosis? a. Atenolol b. Spironolactone c. Digoxin d. Amiodarone e. Atorvastatin d. Amiodarone Pulmonary disease caused by medication is well recognised. Amiodarone is a frequently prescribed medication and a well known cause of pulmonary fibrosis. Amiodarone is a class III anti-arryhthmic drug commonly used in heart failure patients. Its side effects include fatal interstitial pneumonitis, diffuse alveolar damage and pulmonary fibrosis. Other drugs which are known to cause pulmonary fibrosis include busulfan, bleomycin, methysergide and cyclophospamide. These drugs are largely used in the treatment of patients with cancer, rather than the cardiac arena. Of the other drugs in the list above, the only other one which has an important effect on the respiratory system, is atenolol (a beta-blocker). These agents are relativelycontra-indicated in asthmatics since they can cause broncho-constriction.

Page 23: 80 Neuro Resp Gastro Haem Questions

www.AceMedicine.com

Respiratory SBA A 45 year old oil rig worker is admitted with fever and shortness of breath following a return from a secondment overseas. Four days into his admission he develops worsening symptoms and a chest radiograph reveals a pleural collection. A pleural tap is performed due to concerns this may represent an empyema. Which of the following results is most fitting with an empyema?

a. Protein 2.4 g/dl LDH 378 mmol/l pH 7.15

b. Protein 3.8 d/dl LDH 370 mmol/l pH 7.15

c. Protein 2.8 g/dl LDH 277 mmol/l pH 7.45

d. Protein 3.8 g/dl LDH 294 mmol/l pH 7.45

e. Protein 3.4 g/dl LDH 213 mmol/l pH 7.38

b. Protein 3.8 g/dl LDH 370 mmol/l pH 7.15 A pleural effusion is a collection of fluid in the pleural space, between the parietal and serosal layers of the pleura. Pleural aspiration is usually necessary to determine the cause and nature of the effusion, whereby a sample is sent for biochemical analysis, bacteriological culture and microscopic examination. The effusion can then be classified into a transudate or exudate depending on its biochemical characteristics. pH is measured when there is concern regarding an empyema as it typically has a pH of less than 7.2. Transudates have a protein content of <3g/dl and a lactate dehydrogenase (LDH) concentration of <200mmol/l. Causes of transudates include: cardiac failure, hypoalbuminaemia (‘nutritional failure’), renal failuire, liver failure and constrictive pericarditis. Exudates have a protein content of >3g/dl and a LDH concentration of >200mmol/l. Causes of exudates include: lobar pneumonia, lung cancer, mesothelioma, pulmonary embolus, TB and empyema. Empyemas are infected pleural collections which require prompt drainage. This is attempted in the first instance with insertion of a pleural drain., If drainage fails, surgical intervention is required. Empyemas have a low pH (< 7.2), are exudates (protein > 3g/dl) and have a high LDH content (> 200 mmol/l).

Page 24: 80 Neuro Resp Gastro Haem Questions

www.AceMedicine.com

Respiratory SBA A 68 year old pub landlord and lifelong smoker is admitted with lethargy and drowsiness. Laboratory tests reveal results consistent with the syndrome of inappropriate anti-diuretic hormone secretion (SIADH) and a chest radiograph demonstrates a 3cm right upper lobe mass. He is due to undergo a CT guided biopsy of the mass. What is the histological subtype most likely to be? a. Broncho-alveolar cell carcinoma b. Squamous cell carcinoma c. Pulmonary hamartoma d. Ovarian carcinoma metastasis e. Small (oat) cell carcinoma E. Small (oat) cell carcinoma SIADH involves the inappropriate secretion of ADH which leads to retention of water and hyponatraemia. SIADH may be caused by many tumours, such as prostate, thymus, pancreatic and lymphomas, however the most common tumour causing SIADH are small cell carcinomas of the lung. Small cell carcinomas (oat cell carcinomas) often originate from APUD cells (amine precursor uptake decarboxylase cells- neuroendocrine cells). There is a high occurrence of paraneoplastic syndromes associated with this type of tumour, so presentation can be very varied.

Page 25: 80 Neuro Resp Gastro Haem Questions

www.AceMedicine.com

Respiratory SBA A 36 year old woman is admitted with shortness of breath and pleuritic chest pain following a 16 hour flight to London from Borneo. The working clinical diagnosis is of pulmonary embolus and a CT-pulmonary angiogram is requested. What is the most important consideration before the test can go ahead? a. Normal liver function tests b. Normal serum glucose c. Normal renal function d. Normal thyroid function e. If she is on the oral contraceptive pill c. Normal renal function A CT pulmonary angiogram (CTPA) is a diagnostic test employed to image the pulmonary arteries, its main use is to diagnose pulmonary embolism. In order to carry out a CTPA, contrast is administered intravenously. Radiological contrast media are nephrotoxic. This effect is dose dependent, and in many patients the effect is mild, transient, fully reversible and of no clinical significance. The risk of contrast nephropathy is amplified by the presence of coexisting conditions, which include: Renal impairment Hypovolaemia Low cardiac output Diabetes mellitus (especially if treated with metformin) Hyperviscosity (myeloma). For this reason, and in answering the question above, one MUST conduct a renal function test prior to requesting a CTPA (or other contrast imaging investigation), in order to prevent contrast nephropathy. An estimated glomerular filtration rate (eGFR) is probably most useful in most cases. One of the rights bestowed on a newly qualified doctor is that of a referrer for imaging investigations. An essential element of this is to check the preconditions, such as renal function, before requests are made.

Page 26: 80 Neuro Resp Gastro Haem Questions

www.AceMedicine.com

Respiratory SBA A 71 year old man with a long history of COPD is admitted for the 8th time this year with shortness of breath. He is on maximum medical therapy, including home oxygen. An arterial blood gas is performed which is interpreted as showing type 2 respiratory failure. Which of the results is most consistent with that diagnosis? a. pH 7.55 PaO2 7.1 kPa PaCO2 6.1 kPa Bicarbonate 34.8 mmol/l b pH 7.45 PaO2 8.4 kPa PaCO2 7.1 kPa Bicarbonate 29.4 mmol/l c pH 7.45 PaO2 7.2 kPa PaCO2 6.4 kPa Bicarbonate 34.1 mmol/l d. pH 7.27 PaO2 7.2 kPa PaCO2 6.9 kPa Bicarbonate 34.8 mmol/l e pH 7.27 PaO2 7.1 kPa PaCO2 5.9 kPa Bicarbonate 34.6 mmol/l d. pH 7.27 PaO2 7.2 kPa PaCO2 6.9 kPa Bicarbonate 34.8 mmol/l COPD (chronic obstructive pulmonary disease) has become the most popular term to describe patients with spectrum of chronic bronchitis and emphysema. Pulmonary function tests commonly show the following results: Increased residual volume and total lung capacity Reduced vital capacity, FEV1, peak expiratory flow rate, and FEV1:FVC ratio. Respiratory failure is defined as a PaO2<8kPa (60mmHg), and is divided into type I and type II. In type I respiratory failure, PaCO2<6.5kPa. PaO2 is low (hypoxaemic) but PaCO2 may be normal or low. In type II respiratory failure, the PaCO2>6.5kPa and the PaO2 is low. The significance of this classification is that in type II respiratory failure the patient may have developed tolerance to increased levels of PaCO2: in other words, the drive for respiration no longer relies on hypercapnic drive (high PaCO2), but instead on a hypoxic drive (low PaO2). Therefore if the patient is given a high concentration of oxygen therapy, the hypoxic drive for ventilation may decrease. Respiratory EMQ: CAVITATING LUNG LESIONS a. Squamous cell carcinoma metastasis b. Squamous cell primary lung malignancy c. Wegener’s granulomatosis d. Tuberculosis e. Pulmonary infarct f. Staphylococcus pneumonia g. Progressive massive fibrosis h. Aspergilloma A 68 year year old retired British Coal employee with a long history of shortness of breath attends a respiratory outpatient clinic. This is his yearly clinic review at which he has a chest x-ray. His

Page 27: 80 Neuro Resp Gastro Haem Questions

www.AceMedicine.com

symptoms are unchanged from 12 months previously. The report of his CXR reads ‘multiple cavitating lesions in both upper lobes, which are unchanged from the previous film’. 1: Progressive Massive Fibrosis (G) Progressive massive fibrosis (PMF) is due to progression of Coal worker’s pneumoconiosis. With coal worker’s pneumoconiosis, the chest x-ray shows small pulmonary nodules less than 1.5mm. With progressive massive fibrosis, the chest x-ray reveals large fibrotic masses (1-10 cm), predominantly in the upper zones. These masses may cavitate over time. A 62 year old man with a 10 year history of emphysema admits to increasing shortness of breath over the past 3 weeks. Over the last few days he has had several episodes of haemoptysis. He is known to have several upper lobe bullae. His chest x-ray report reads, ‘Air crescent sign within a cavitating right upper lobe lesion, correlating with one of the established bullae’. 2: Aspergilloma (H) Aspergilli are typically inhaled as small spores which do not affect people without underlying lung or immune system disease. However, people with pre-existing lung problems, especially cavitating lung diseases, are at risk. This is because the fungus can settle in the cavities within the lungs, and is able to grow free from interference as the immune system is unable to penetrate the cavity. The chest x-ray findings show lesions with an air “halo/crescent” around them.People can often be unaware for some time before incidental diagnosis. However, a small percentage of aspergilliomas invade into the cavity wall and can result in bleeding and hence haemoptysis, which may require resection of the damaged lung area containing the aspergilloma. A 35 year old man with a 3 month history of sinus disease for which he is attending an ENT clinic is admitted with increasing shortness of breath. His chest x-ray reveals multiple cavitating lungs lesions in both lungs. His weight is stable and his white blood cell count is normal. 3: Wegener’s granulomatosis (C) The patient’s clinical presentation is most in keeping with Wegener’s granulomatosis. This is a multisystem disorder due to a granulomatous reaction within small/medium sized arteries and veins. It typically starts with rhinorrhoea, with subsequent nasal mucosal ulceration, cough, dyspnoea, haemoptysis, epistaxis and/or pleuritic pain. A “saddle nose” deformity is also a feature sometimes seen in this condition. The chest x-ray shows nodular masses or pneumonic infiltrates with cavitation. This disease responds well to immunosuppression, eg. with cyclophosphamide. A 17 year old 1st year university student attends her GP with a short history of shortness of breath and a productive cough. She has been ignoring her symptoms with all the excitement of university and life in halls of residence, but it is now restricting her activities of daily living. A chest x-ray reveals a 3 cm cavitating left upper lobe lesion, with an enlarged left hilum. 4: Tuberculosis (D) Tuberculosis transmission is through air and direct contact, whereby the initial infection is with Mycobacterium tuberculosis. Primary TB is usually asymptomatic; however,

Page 28: 80 Neuro Resp Gastro Haem Questions

www.AceMedicine.com

occasionally there may be erythema nodosum, a small pleural effusion or pulmonary collapse caused by compression of a lobar bronchus by enlarged nodes. The chest x-ray typically shows consolidation, air space or nodular changes in the upper zones, with loss of volume of the lung, and an enlarged hilum due to the enlarged lymph nodes. A 67 year old life-long smoker attends his GP with hoarseness and weight loss. He has several palpable nodes bilaterally within the anterior triangle of the neck. In the first instance, a chest x-ray is performed which reports multiple bilateral cavitating lung lesions measuring up to 2cm. 5: Squamous cell carcinoma metastasis (A) Sqaumous cell carcinoma is the commonest malignant tumour of the larynx affecting both men and women in a ratio of 5:1. The disease accounts for 1% of all male malignancies. Incidence increases with age, with the peak occurring at 60-70 years. Patients often present with hoarseness, although dyspneoa and stridor are late signs. Predisposing factors include alcohol and tobacco smoke (the condition is very rare in non-smokers). It must be noted that the squamous cell primary lung malignancy is a possible differential diagnosis, but due to the multiple palpable lymph nodes, answer A is most likely. Respiratory EMQ : INTRATHORACIC MALIGNANCIES A. Bronchoalveolar cell carcinoma B. Squamous cell carcinoma C. Small cell carcinoma D. Pleural fibroma E. Mesothelioma F. Thymoma G. Teratoma H. Large cell carcinoma A 56 year old smoker presents to his GP with lethargy and a change in appearance including an increase in weight, predominantly around the trunk. (C) Small cell carcinoma (ACTH secreting) Small cell carcinomas are also commonly known as oat cell carcinomas, and account for 20-30% of all lung cancers. They arise from endocrine cells explaining why polypeptide hormones are often secreted by these tumours. These enhance tumour growth and result in paraneoplastic syndromes, such as ectopic production of ACTH (seen in this case) which produces an atypical type of Cushing’s syndrome. Typical features include weight loss, pigmentation, metabolic acidosis, hyperglycaemia, and hypertension. The classic features of Cushing’s are often absent. Oat cell carcinomas are typically found in middle aged male smokers. Although the tumours are normally rapidly growing

Page 29: 80 Neuro Resp Gastro Haem Questions

www.AceMedicine.com

and highly malignant, it is the only bronchial carcinoma that responds well to chemotherapy. A 67 year old former electrician is admitted to hospital with increasing shortness of breath. He is a lifelong non-smoker. Clinically he has a large left sided pleural effusion which, when drained, was bloody and laboratory analysis identified an exudate. Bronchoscopy was normal. (E) Mesothelioma Mesothelioma is a malignant tumour of the visceral or parietal pleura, which may present as discrete pleural deposits or as a localized lesion. It is usually associated with asbestos exposure, however exposure may have been light and there may be a 20-40 year interval from exposure to disease. On x-ray the common finding is a unilateral pleural effusion and pleural thickening, with bloody pleural fluid being identified on tapping. Clinical features include pleuritic pain with increasing dyspneoa, weight loss, finger clubbing, and recurrent pleural effusions. A 65 year old lady is admitted with episodes of syncope Her fasting blood sugar is 2.6 mmol/l. Respiratory examination is normal. The chest x-ray is abnormal but there is no mediastinal abnormality. (D) Pleural fibroma Pleural fibroma is a rare and benign primary tumor, which is mainly asymptomatic. It is not associated with asbestos exposure. The solitary mass can grow to be very large due to its slow growth from the visceral pleural. Pleural fibromas most commonly affect women with the mean age of presentation being around 50 years of age. These tumours generally behave in a benign fashion, although malignant tumours do exist. These tumours are known to be able to produce insulin-like growth factor II (IGF-II), causing the increased utilization of glucose, inducing the hypoglycaemia seen here. A 25 year-old man attends an outpatient clinic with lethargy and mild stridor. Chest x-ray reveals an anterior mediastinal mass, which is confirmed on CT and shown to contains areas of fat and calcification. (G)Teratoma Teratomas are neoplasms of germ cell origin. They occur most often in the gonads, where germ cells are abundant. Teratomas can occasionally arise elsewhere in the body, usually in the midline, possibly from germ cells that have been arrested in their migration. These extra-gonadal sites for teratomas include the mediastinum and sacro-coccygeal region. The mass in this patient’s case may produce mild stridor due to a degree of obstruction of the trachea. A 57 year-old man attends his GP complaining of increasing shortness of breath and myalgia. His GP refers him to rheumatology, who request a battery of tests including a chest radiograph and an

Page 30: 80 Neuro Resp Gastro Haem Questions

www.AceMedicine.com

‘auto-antibody screen’. This reveals an anterior mediastinal mass and a positive Anti-AChR (Anti-acetylcholine receptor) antibody. (F)Thymoma Thymomas may be associated with myasthenia gravis whereby the anti-body mediated disease causes muscle fatigue due to the depletion of functioning post-synaptic receptor sites. In 10% of myasthenia gravis patients, a thymic tumour is found, the incidence increasing with age. Fatiguability is the single most important feature, affecting the proximal limb muscles, extraocular muscles, and the muscles of mastication, speech and facial expression. Respiratory difficulties may also occur. Preliminary tests may show a thymoma on chest x-ray that may be confirmed by cross sectional imaging with CT. Respiratory EMQ :RESPIRATORY BASED INVESTIGATIONS a. Chest x-ray b. CT pulmonary angiogram c. Pulmonary function tests d. Arterial blood gas sampling e. High resolution CT scan of chest f. Bronchoscopy g. Transbronchial biopsy h. Positron emission tomography (PET) scan (including chest) A 69 year old smoker has been diagnosed with a stage T1, N1, M0 right upper lobe bronchial carcinoma. He has a good co-morbid status and is a potential candidate for a curative right upper lobectomy. (H) Positron emission tomography (PET) scan (including chest) Bronchial carcinoma with a stage of T1, N1, M0 indicates that there lesion in less than 3cm and not involving the bronchus or pleura (T1), has ipsilateral hilar node metastases (N1) and no distant metastatasis (M0). Positron emission tomography (PET) scanning is being increasingly used in the diagnosis and staging of lung cancer. It may detect metastatic disease in the mediastinal nodes, even if they are not enlarged, and is a more accurate imaging technique in the staging of lung cancer, when major curative surgery is being considered. A 50 year old lady is admitted with shortness of breath, haemoptysis and right sided pleuritic chest pain. Her chest x-ray is normal. D-dimer is elevated.

Page 31: 80 Neuro Resp Gastro Haem Questions

www.AceMedicine.com

(B) CT pulmonary angiogram This patient’s current symptoms, together with her increased d-dimer, suggests a diagnosis of pulmonary embolism. In order to diagnose this, a CTPA (computed tomography pulmonary angiogram) is required. This involves the injection of iodinated contrast medium into a peripheral vein, in order to rapidly image the pulmonary vessels as the contrast is injected. It is a very sensitive test and has superseded V/Q scanning in most centres. A 44 year old man with increasing shortness of breath and lethargy has a chest radiograph which demonstrates that the lungs are clear. However, there is right paratracheal lymphadenopathy. Serum ACE is elevated. A presumptive diagnosis of sarcoidosis is made for which treatment commencement is being considered. (C) Pulmonary function tests Sarcoidosis may have various clinical features, including bilateral hilar lymphadenopathy, erythema nodosum, pulmonary fibrosis, elevated serum ACE levels and more. The diagnosis of sarcoidosis can often be made on clinical grounds, particularily when a young adult presents with classical features such as the bilateral hilar lymphadenopathy and erythema nodosum. Although a tissue biopsy would be diagnostic, lung function tests are often conducted first as they are less invasive. The lung function tests may be normal or show reducing lung volumes, impaired gas transfer and a restrictive ventilatory defect due to progression of the disease causing fibrosis of the lungs. Pulmonary function tests may then be performed and reviewed following steroid therapy to access response to treatment. A 45 year old patient with rheumatoid disease has complained of increasing shortness of breath over the past 9 months. 1 year previously her disease modifying anti-rhematoid drug was changed to methotrexate. (E)High resolution CT scan of chest It is known that methotrexate may induce lung disease such as pulmonary fibrosis. With interstitial lung diseases such as those caused by methotrexate, high resolution CT (HRCT) imaging is the ideal imaging modality. As HRCT's aim is to assess a generalized lung disease, the test is conventionally performed by taking thin sections 10-40 mm apart. The result is a few images which should be representative of the lungs in general, but which cover only approximately one tenth of the lungs. Because HRCT does not image the whole lungs (by using widely spaced thin sections), it is unsuitable for the assessment of lung cancer or other localised lung diseases. A 17 year old tall man becomes acutely short of breath following an aggressive collision whilst playing in the local football league. (A)Chest x-ray In high impact trauma cases, the likelihood of a patient suffering from a pneumothorax is an important consideration.

Page 32: 80 Neuro Resp Gastro Haem Questions

www.AceMedicine.com

Respiratory EMQ: SOLITARY PULMONARY LESION a. Bronchial carcinoma b. Round pneumonia c. Lung abscess d. Pulmonary infarct e. Arteriovenous malformation f. Bronchogenic cyst g. Pulmonary harmatoma h. Aspergilloma A 17 year old woman presents with shortness of breath and raised inflammatory markers. A chest x-ray taken 3 months previously was normal. There is now a 3 cm non-cavitating lesion in the left lower lobe. (B)Round pneumonia Round pneumonia affects children more commonly than adults. It is a spherical pneumonia caused by Haemophilus influenza or Streptococcus (pneumococcus). Its location is usually in the lower lobe, most often posterior. Its clinical features include cough, fever, chest pain, malaise, dyspnoea and haemoptysis. A chest x-ray should be repeated 6-8 weeks after clinical recovery to confirm resolution. A 32 year old longstanding asthmatic with recurrent hospital admissions is admitted with fever and frank haemoptysis. A chest x-ray shows a 3cm cavitating lesion in the left upper lobe with an ‘air crescent’ within it. No mediastinal lymphadenopathy (H)BAspergilloma This is a totally separate disease from allergic bronchopulmonary aspergillosis. It simply represents the growth within previously damaged lung tissue, of A. Fumigatus, which forms a ball of myecelia within lung cavities. The typical appearance on chest x-ray is of a round lesion with an ‘air crescent’ above it. The aspergilloma itself causes little trouble, though occasionally significant haemoptysis may occur due to erosion into an adjacent vessel, requiring resection of the damaged area of lung containing the aspergilloma. A 44 year old man has a screening chest x-ray for work placement in Australia. He has a 10 pack-year history of of smoking. He is asymptomatic. Chest x-ray shows a 2cm right upper lobe nodule with calcification within. (G) Pulmonary harmatoma This is the most common benign tumour of the lung and is usually seen on chest x-ray as a very well defined round lesion 1-2 cm in diameter in the periphery of the lung. They occur as solitary round nodules with no predilection for any lobe. Growth is extremely slow, but the tumour can eventually

Page 33: 80 Neuro Resp Gastro Haem Questions

www.AceMedicine.com

reach several centimeters in diameter. Harmatomas are more common in men and in smokers. Peripheral nodules are asymptomatic, although endobronchial lesions are frequently associated with symptoms or signs of obstruction. As such, they must be distinguished from malignant neoplasms or recognized as the cause of a pneumonia or atelectasis. They usually contain fat and ‘popcorn’ calcification. A 27 year old asymptomatic lady has a screening chest x-ray for her new employment. There is a single right middle lobe lesion on chest x-ray. Her only medical history is of admission to hospital with a brain abscess 6 months ago. (E) D- Arteriovenous malformation Despite the presumed congenital origin of AVMs, the clinical presentation most commonly occurs in young adults. Pulmonary ateriovenous malformations are more common in females and may present with primarily neurologic manifestations when pulmonary symptoms are absent or unrecognized. Brain abscess, embolic stroke, and hemorrhage from concomitant brain AVMs are well-recognized complications. Whenever a pulmonary AVM is suspected, the presence of a right-to-left shunt should be confirmed by the performance of a 100% oxygen study, contrast-enhanced echocardiography, or radionuclide perfusion lung scanning. Hereditary haemorrhagic telangectasia is stongly associated with AVMs. A 19 year old student has a chest x-ray before his year overseas for his university course. Chest x-ray shows a 3cm left lower lobe. Old images are ascertained from 7 years ago showing an identical unchanged abnormality. Bronchogenic cyst (F) Bronchogenic cysts may result from abnormal budding of the tracheobronchial tree. They are lined by bronchial elements such as cartilage, smooth muscle and ciliated respiratory epithelium. They are classified by their position - central or peripheral. Cysts are usually single, spherical or oval, unilocular masses. They are mainly asymptomatic and can present at any age, although they are more common in men. Surgical excision is recommended. Radiologically, it is impossible to differentiate a bronchogenic cyst and malignancy.

Page 34: 80 Neuro Resp Gastro Haem Questions

www.AceMedicine.com

Gastroenterology MCQ 1 A 35 year old man has a short period of fatigue and myalgia. His GP performs a number of basic blood tests, including liver function tests. Bilirubin 22 µmol/l ALT 101 IU/L AST 113 IU/L ALP 43 U/L GGT 44 IU/L Which of the following conditions are potential causes of the liver function test abnormalities?

A. Cholangiocarinoma B. Prescribed medications C. Viral infection D. Pancreatic carcinoma E. Autoimmune hepatitis

F,T,T,F,T Deranged liver function tests (LFTs) can be broadly divided into hepatocellular (hepatitic), obstructive or mixed pictures. With a hepatocellular picture, the AST and ALT are the predominantly elevated liver enzymes. This can be due to a wide range of causes, including autoimmune hepatitis, alcohol, fatty liver disease, viral infections and from a number of prescribed medications, in particular antibiotics. In obstructive cases the bilirubin, ALP and GGT tend to be elevated. Causes include pancreatic carcinoma, biliary strictures, bile duct calculi and cholangiocarcinoma. Gastroenterology MCQ 2 A 48 year old man with a history of ulcerative colitis attends the gastroenterology clinic for his regular review. Over the past few months his bowel symptoms have been well controlled, but he has suffered from itching and fatigue. The following blood test results are ascertained. Na+ 144 mmol/l K+ 4.2 mmol/l Urea 5.9 mmol/l

Creatinine 70 mol/l Hb 13.1 g/dl WCC 5.1 x109/l PLT 278 x109/l Autoantibody screen: Perinuclear anti-neutrophil cytoplasmic antibody (pANCA) positive, Anti-nuclear antibody (ANA) positive Anti-mitochondrial antibody (AMA) negative

Bilirubin 65 mol/l

Page 35: 80 Neuro Resp Gastro Haem Questions

www.AceMedicine.com

ALT 75 U/l AST 64 U/l ALP 290 U/l GGT 300 U/l Which of the following is the most likely diagnosis?

A. Cholelithiasis B. Hepatocellular carcinoma C. Primary sclerosing cholangitis (PSC) D. Viral cholangitis E. Primary biliary cirrhosis (PBC)

C. Primary sclerosing cholangitis (PSC) Primary sclerosing cholangitis is a chronic liver disease resulting from inflammation of the bile ducts, which is associated strongly with ulcerative colitis. The GGT and ALP levels are usually the most elevated of the liver enzymes although rises in the tranaminases (AST and ALT) can occur and bilirubin in more advanced disease. The disease is believed to be autoimmune in nature, hence both anti-nuclear antibody (ANA) and perinuclear anti-neutrophil cytoplasmic antibodies (pANCA) may be positive. Primary biliary cirrhosis is another autoimmune liver disease, in which ANA may be positive, but more typically anti-mitochondrial antibodies (AMA) are also present. Gastroenterology SBA 1) Nutrition A 64-year-old man had been in ITU for 11 days. He was admitted with a ruptured aortic aneurysm that was successfully repaired, but currently remains ventilated. He had a feeding nasogastric tube inserted 6 days ago, but large amounts of gastric contents were still being aspirated. The best way to feed this patient would be to: A) Convert to intravenous saline and dextrose B) Insert a percutaneous endoscopic gastrostomy (PEG) tube C) Persist with nasogastric feeding D) Start total parenteral nutrition (TPN) E) Try oral feeding, but only with liquids D) Start total parenteral nutrition (TPN) This patient has been in ITU for a prolonged time and has a problem with feeding most likely due to an ileus. Gastric stasis is a common post operative problem. He had a NG tube inserted as he is still being ventilated. He will need some form of nutrition as the current method of nasogastric (NG) feeding is not working – this is because whatever is being passed into his stomach is being aspirated

Page 36: 80 Neuro Resp Gastro Haem Questions

www.AceMedicine.com

straight back up again (current standard practice in ITU). If NG feeding is not working then attempting oral feeding even with liquids is bound to fail as well. Inserting a PEG is not a valid option because the problem is with the stomach not shifting the feed that is being put into it – PEG tubes allow feeding into the stomach via the anterior abdominal wall. In addition PEG tubes are not used for temporary measures such as an ileus – it is reserved for patients who have chronic conditions, such as stroke where they are unable to swallow safely. To persist with intravenous fluids such as saline and dextrose is unlikely to match his nutritional needs and will inevitably make him extremely oedematous. This patient needs some form of nutrition that does not involve his stomach. There are 2 options: a) persist with the enteral route, but insert a naso-jejunal tube so that the feed goes beyond the stomach b) start parenteral nutrition as this avoids using the gut completely. Most of the time TPN is given for 1-2 weeks and then hopefully the patient’s gut will start working again. Gastroenterology SBA 2) Viral Hepatitis A 45-year-old man is found to have deranged liver function tests by his GP. As part of the investigations, the GP requested viral hepatitis serology: Hepatitis A antibody Negative Hepatitis B core antibody Positive Hepatitis B surface antigen Negative Hepatitis C antibody Positive The following conclusion that can be made about the patient is: A) He is a carrier for hepatitis A B) He is a carrier for hepatitis B C) He is a carrier for hepatitis C D) He has been vaccinated against hepatitis B E) He does not have viral hepatitis C) He is a carrier for hepatitis C Hepatitis A (HAV) and hepatitis E (HEV) are hepatitis viruses that do not have a chronic carrier state – you either survive the infection or will die from it. If someone is HAV antibody positive then it means that they have had the infection in the past but have survived it but are not currently infected with it. Hepatitis B (HBV), hepatitis C (HCV) and hepatitis D (HDV) can cause chronic infection (although HDV can only occur in the presence of ongoing HBV infection). HBV infection is diagnosed by looking for the presence of surface antigen (HBsAg). If HBsAg is positive then the patient has HBV infection – if this is present for more than 6 months the patient is said to have chronic HBV infection. HBV core antibody (HBcAb) gives an indication of previous infection or natural immunity. In this case, the patient has had HBV infection in the past but has managed to clear it. Individuals who are given HBV

Page 37: 80 Neuro Resp Gastro Haem Questions

www.AceMedicine.com

vaccination are given a recombinant version of HBsAg and NOT core antibody or core antigen (HBcAb or HBcAg). If you are HBcAb positive (like this patient), then there is no benefit from giving the patient additional HBV vaccination. Patients who are immune from HBV, either through active or passive immunity, will be HBV surface antibody (HBsAb) positive.

Interpretation HBcAb (HBV core antibody)

HBsAg (HBV surface antigen)

HBsAb (HBV surface antibody)

Active immunization Negative Negative Positive

Passive immunization (previous infection now cleared)

Positive Negative Positive

Chronic HBV infection Positive Positive Negative

Unlike HBV, the serological diagnostic test for HCV is the presence of antibody in the blood; hence if you are HCV antibody positive then you are infected with HCV as is the case here. The next test to be done would now be HCV PCR RNA to characterize the HCV viral load (i.e. how much virus the patient has) and also the genotype, because this will affect treatment duration and prognosis. Gastroenterology SBA 3) Colorectal Carcinoma Which one of the following conditions is NOT associated with an increased risk of colorectal cancer ? A) Familial adenomatous polyposis (FAP) B) Osler-Weber-Rendu syndrome C) Peutz-Jegher syndrome D) Primary sclerosing cholangitis (PSC) E) Ulcerative colitis (UC) B) Osler-Weber-Rendu syndrome FAP is an inherited condition where there are >100 adenomatous polyps in the colon and has a high malignant potential. It is also referred to as Gardner's syndrome, if there are extra-colonic lesions. These include benign lesions such as: osteomas; dental abnormalities like super-numerary teeth; skin lesions like epidermoid cysts, sebaceous cysts, fibromas and lipomas; adrenal adenomas and nasal angiofibromas. Malignant lesions include duodenal, periampullary, thyroid, pancreatic, and gastric tumours; hepatoblastomas; and tumours of the central nervous system. Peutz-Jegher is another inherited condition where there may be hyperpigmentation of the mouth and limbs along with development of hamartomatous polyps throughout the gastrointestinal tract.

Page 38: 80 Neuro Resp Gastro Haem Questions

www.AceMedicine.com

Both UC and Crohn's are forms of inflammatory bowel disease (IBD) and have an increased risk of developing colorectal cancer. The risk develops after 7 years of disease duration and depends on the extent of bowel involvement and the severity of bowel inflammation. Over 70% of patients with PSC have underlying UC and are at risk of cholangiocarcinoma and colorectal cancer; it is for this reason that they should have annual bowel colonoscopic screening. Osler-Weber-Rendu syndrome or Hereditary Haemorrhagic Telangiectasia (HHT) is an autosomal dominant condition associated with epistaxis; cerebral, gastrointestinal, hepatic and pulmonary arterio-venous malformations (AVM) and muco-cutaneous telangiectasias that can cause bleeding; and iron deficiency anaemia. They are not associated with cancer. Gastroenterology SBA 4) Paracetamol Overdose A 18-year-old female presented to A&E with abdominal pain and nausea. She admitted to taking 40 grams of paracetamol with a bottle of wine over the past 2 days. She was remorseful of her actions and now wanted treatment. On examination she was alert and orientated. Her temperature was 37.5, pulse 98 bpm and blood pressure was 120/74 mmHg. There was generalised abdominal tenderness, but no signs of chronic liver disease. The most appropriate treatment for her condition is: A) Intravenous Flumazenil B) Intravenous Fresh frozen plasma (FFP) C) Intravenous Naloxone D) Intravenous N-Acetyl Cysteine (Parvolex) E) Intravenous Thiamine (Pabrinex) D) Intravenous N-Acetyl Cysteine (Parvolex) The standard management for paracetamol overdose is to wait 4 hours post overdose and then determine the blood paracetamol level. According to the nomogram found in A&E departments and the British National Formulary, if the level is below the treatment line then no treatment is necessary. There is a separate nomogram for high risk patients who might have taken alcohol or other sedatives – the principle is still the same in that if the paracetamol level is below this “high risk” treatment line, then no medical treatment is necessary. If the patient’s level is above the treatment line, then the patient should be admitted and given N-Acetyl Cysteine (Parvolex) for at least 3 days or until the INR returns to normal. The INR will peak at day 3 post paracetamol overdose and as the INR is used as one of the indicators for referral to a liver transplant unit, it is vital that fresh frozen plasma (FFP) should not be given unless the patient is actively bleeding. The other indicators for referral include the presence of encephalopathy, renal failure, severe thrombocytopaenia and acidosis that does not respond to fluid resuscitation. In this case, there is no accurate time for when the overdose occurred because the patient “staggered” the overdose over 2 days as well as took alcohol. Therefore, the treatment nomogram

Page 39: 80 Neuro Resp Gastro Haem Questions

www.AceMedicine.com

has no role to play and the patient should be treated with N-Acetyl Cysteine straight away for at least 3 days until the INR returns to normal. Flumazenil is an antidote for benzodiazepine overdose. Naloxone is an antidote for opiate overdose. Thiamine (Pabrinex) is used to treat patients with alcoholic liver disease because they are usually nutritionally deficient in vitamin B1; giving intravenous dextrose may precipitate Wernicke’s encephalopathy, which is why such patients are given Pabrinex first. Gastroenterology SBA 5) Chronic Pancreatitis A 49-year-old man was admitted to A&E with central abdominal pain that was radiating into his back. He has had this current pain for the past 3 days, but in the past he had had a number of previous attacks which required hospital admission. He also complained of a 6 kg weight loss over the past 4 months. In addition to this, he also mentioned that he was passing offensive bowel motions that were difficult to flush. He used to be a heavy alcohol drinker, but was currently abstinent. Recently he had been diagnosed with type 2 diabetes. On examination he looked thin, but was haemodynamically stable. There was generalised abdominal tenderness but no guarding. The most likely diagnosis is: A) Cholecystitis B) Chronic pancreatitis C) Colonic carcinoma D) Decompensated alcoholic liver disease E) Peptic ulcer B) Chronic pancreatitis This patient has central abdominal pain with steatorrhoea and diabetes. All of these would fit in with chronic pancreatitis which has developed after recurrent attacks of acute pancreatitis. The steatorrhoea is due to loss of exocrine function and malabsorption. The diabetes is most likely due to loss of endocrine function. Weight loss can occur because these patients have problems with malabsorption, but one should always be concerned about developing pancreatic carcinoma, of which chronic pancreatitis is a risk factor. Cholecystitis typically presents with upper abdominal (and specifically right upper quadrant pain), fever and raised white cell count in the presence of an inflamed gall bladder. Biliary colic is pain due to gallstones trying to pass through the cystic duct and occurs when the gall bladder contracts in response to a fatty meal. It occurs in the absence of an inflamed gall bladder but the pain can be similar; however, unlike biliary colic, cholecystitis won't spontaneously resolve completely due to the underlying inflamed gall bladder. Peptic ulcer disease also causes upper abdominal (and specifically epigastric) pain, often referred to as dyspepsia, but may or may not have any relation to eating. It can also cause reflux, and if severe, can be associated with upper gastrointestinal haemorrhage that may present as haematemesis or

Page 40: 80 Neuro Resp Gastro Haem Questions

www.AceMedicine.com

melaena. In clinical practice it is often not possible to differentiate between gastric or duodenal ulceration/ inflammation, with the diagnosis being made endoscopically. They are best treated with proton pump inhibitors. The main risk factors for developing peptic ulcer disease are NSAID use. Colonic carcinoma can present acutely as an emergency usually as bowel obstruction, an acute abdomen due to intestinal perforation (and therefore presenting with guarding), or with rectal bleeding. Decompensated liver disease is a possibility but does not usually present with abdominal pain. Typically, it would present with the complications of cirrhosis: jaundice; encephalopathy presenting as confusion or decreased conscious state; variceal haemorrhage presenting as haematemesis, melaena, or rectal bleeding; or ascites presenting as a distended abdomen due to fluid accumulation in the peritoneal cavity. Gastroenterology SBA 6) Ascites A 61-year-old woman with known alcoholic liver disease was referred by her GP with increased abdominal distension. Her weight had increased by 5 kg over the past week. She used to drink 30 units per week, but had now stopped drinking alcohol. She was not on any medication. On examination she was apyrexial and was not in distress. Her pulse was 88 bpm, regular and blood pressure was 110/78 mmHg. She had multiple spider naevi and moderate abdominal distension with obvious shifting dullness. Routine blood tests have revealed no electrolyte or clotting abnormalities. The most appropriate treatment for this patient is: A) Ciprofloxacin B) Diazepam C) Digoxin D) Spironolactone E) Warfarin D) Spironolactone A patient with abdominal distension in the presence of chronic liver disease (spider naevi and history of heavy alcohol intake) has probably developed ascites. Whilst the exact mechanism of why ascites develops is not fully understood, it tends to occur in patients with portal hypertension and hypoalbuminaemia. There is avid sodium and water retention in cirrhosis - if there is decreased excretion of sodium in the urine compared to dietary intake of sodium, then ascites +/- peripheral oedema will eventually occur. This is why the mainstay of treating chronic liver disease is to restrict dietary sodium intake; fluid restriction is not necessary because fluid follows sodium passively. In this case, additional measures to also increase sodium urinary excretion need to be adopted in the form of diuretics. Spironolactone is an aldosterone antagonist and blocks its effects on the distal tubule and collecting duct of the kidney. Whilst in cirrhosis, it is thought that there is hyperaldosteronism and increased

Page 41: 80 Neuro Resp Gastro Haem Questions

www.AceMedicine.com

activation of the renin-angiotension-aldosterone system; the exact mechanism for why this occurs is not fully understood. This single agent appears to work better than loop diuretics (like furosemide), which work on the ascending limb of the Loop of Henle, and thiazides, which work on the distal tubule. This may be because of loop diuretics and thiazides are protein bound and enter the tubular lumen by secretion in the proximal tubule; it is this process that may be impaired in cirrhosis. The problem with Spironolactone, is that in addition to causing hyponatraemia (like all diuretics), it can also cause hyperkalaemia and so it is important to monitor the renal function and electrolytes in these patients. Spironolactone can be used in conjunction with Furosemide so careful monitoring of the electrolytes is mandatory. If the ascites is too large and is causing respiratory compromise (so called "tense ascites"), then insertion of a drain and performing therapeutic paracentesis can be carried out. Diazepam and other sedative drugs should be avoided in patients with chronic liver disease because of the unpredictable effects of these drugs and may induce hepatic encephalopathy. Ciprofloxacin is an antibiotic that is used in the treatment of spontaneous bacterial peritonitis (SBP). Ascitic fluid, like any other stagnant fluid in the body, has the potential for becoming infected. How this happens is not known but a patient does not necessarily have to have systemic symptoms of feeling unwell. It is for this reason, that ALL patients with ascites should have their ascites tested by a diagnostic tap to exclude SBP. SBP is diagnosed by having ≥250 white cells/mm3 or the growth of organisms from the ascitic fluid. The overall prognosis of patients is much worse once SBP occurs and patients will need to go onto lifelong secondary prophylactic antibiotics (like Ciprofloxacin). Warfarin has no place in the treatment of ascites except possibly if there is a condition causing blockage of the hepatic vein(s) or Budd-Chiari syndrome. Digoxin is used in the treatment of atrial fibrillation. Gastroenterology SBA 7) Wilson's Disease A 20-year-old man was referred by the neurologist to the liver clinic. Two years ago he had developed tremor and rigidity in his upper limbs which was getting steadily worse. He had recently seen the ophthalmologist who had diagnosed Kayser-Fleischer rings. Now he had some derangement of his liver function tests. He does not drink alcohol and has no risk factors for viral hepatitis. On examination he was mildly jaundiced and had hepatomegaly. The most appropriate investigation to establish the diagnosis is: A) Alpha-1 anti-trypsin B) Anti-mitochondrial antibodies C) Autoimmune profile D) Caeruloplasmin E) Iron studies D) Caeruloplasmin This young man has neurological disease and liver disease which is characteristic of Wilson's disease. This is highly likely since there is also the presence of Kayser-Fleischer rings. Wilson's disease is an

Page 42: 80 Neuro Resp Gastro Haem Questions

www.AceMedicine.com

inherited condition where there is a defect in the biliary excretion of copper leading to an accumulation in the liver and brain. Copper binds to apocaeruloplasmin to form caeruloplasmin; with Wilson's disease this binding does not happen and so there is decreased caeruloplasmin detected in the blood. Serum alpha-1 anti-trypsin is used to diagnose alpha-1 anti-trypsin deficiency which is associated with cirrhosis and emphysema. Anti-mitochondrial antibodies are used to test for primary biliary cirrhosis. Iron studies include serum iron, total iron binding capacity (TIBC) and transferrin saturation and are used to screen for haemochromatosis. Gastroenterology SBA 8) Carcinoid Syndrome A 56-year-old woman was referred to the gastroenterology clinic because of watery diarrhoea for the past 4 months. She opens her bowels up to 15 times per day but with no blood per rectum. This is associated crampy abdominal pain but no nausea or vomiting. Her weight has dropped by 7 kilograms in that time. Recently she has developed episodes of facial and neck flushing which could last several minutes, but resolved spontaneously. There is no other previous medical or drug history and she has not travelled abroad recently. On examination, she was apyrexial, but had some generalised abdominal tenderness and 2 cm hepatomegaly.The most likely diagnosis is: A) Carcinoid syndrome B) Colorectal carcinoma C) Cushing's syndrome D) Giardiasis E) Whipple's disease A) Carcinoid syndrome This patient has abdominal pain, diarrhoea and flushing which are the classical features of carcinoid syndrome. Bronchospasm can also occur, but not very often if the tumours occur in the mid-gut which is the commonest location. Carcinoid tumours produce a variety of vasoactive amines (such as 5-HT, noradrenaline and dopamine), peptides (such as bradykinin, vasoactive intestinal peptide and substance P) and prostaglandins which account for the symptoms. These vasoactive products are inactivated by the liver which is why gastrointestinal carcinoid syndrome only occurs when hepatic metastases arise (the likely cause of her hepatomegaly). Whipple's disease is an extremely rare disease characterized by abdominal pain, diarrhoea, weight loss and arthralgia (typically affecting large joints). It is caused by a bacterial infection. Giardia is unlikely in a patient who has not travelled abroad.

Page 43: 80 Neuro Resp Gastro Haem Questions

www.AceMedicine.com

Gastroenterology SBA 9) Coeliac Disease A 20-year-old female had recently been diagnosed with coeliac disease. When she was reviewed in the clinic she was told to keep to a gluten free diet, but she had some concerns about what she could or could not eat. Which ONE of the following should she avoid ? A) Beer B) Corn C) Potatoes D) Rice E) Soybean A) Beer Coeliac disease is a malabsorption syndrome affecting the small bowel which occurs in certain individuals on exposure to dietary gluten. The exact pathogenesis is not fully understood, but it is thought that in susceptible individuals there is the development of antibodies to the gliadin component of gluten. There is a genetic component as there is a 70-100% concordance rate in monozygotic twins and it is associated with HLA DQ2 and DQ8. It can affect all ages and should be screened for in patients with chronic diarrhoea, malabsorption, abdominal distension, bloating and weight loss. Diagnosis is by identifying IgA gliadin, endomyseal or tissue transglutaminase antibodies though there is a risk of false negatives if the patient is IgA deficient. The gold standard test is a small bowel biopsy typically taken from the 2nd part of the duodenum at endoscopy.The mainstay of treatment is maintaining a gluten-free diet. This includes avoiding wheat, oats, rye, malt, brewer's yeast and barley. Rice, beans, potatoes, corn, soya, and legumes don't contain gluten. Beer and lager should be avoided but cider can be consumed. In a small proportion of patients, especially those that do not stick to their gluten-free diet, small bowel lymphoma can occur. Gastroenterology SBA 10) Alcohol Withdrawal A 33-year-old man was brought to A&E after suffering a fit. The fit lasted less than a minute and self terminated. The ambulance crew mentioned that he was acting confused, and appeared to be hallucinating. He mentioned that he was an alcoholic and that this was the second time he was trying to give up alcohol. His last drink of alcohol was 72 hours ago. On examination, he was very agitated, sweating profusely and had a temperature of 38.0. He was disorientated in time and place, but not person, and had a Glasgow Coma Scale of 15/15. His pulse was 135 bpm and regular, with a blood pressure was 180/110 and respiratory rate was 24 breaths per minute. Abdominal examination was normal. Blood glucose was normal. The most appropriate treatment for this patient is:

Page 44: 80 Neuro Resp Gastro Haem Questions

www.AceMedicine.com

A) Diamorphine B) Diazepam C) Haloperidol D) Phenytoin E) Propranolol B) Diazepam Alcohol withdrawal can occur as early as 6-12 hours from the time of the last drink. It starts off with minor withdrawal symptoms (stage 1) of: sweating, tremor, agitation and palpitations and individuals are generally coherent. Following this, major withdrawal symptoms (stage 2) can occur after 24-72 hours and these tend to be more pronounced than with stage 1: confusion, nausea & vomiting, diarrhoea, withdrawal seizures, and sinus tachycardia. Delirium tremens (DT), whilst relatively rare (5-12%), can be potentially fatal and are characterized by: disorientation, drenching sweats, hallucinations, severe tachycardia and hypertension. Death can occur through head injury, cardiovascular complications, aspiration and electrolyte abnormalities. The initial management of this patient would be to first establish he is not hypoglycaemic as this would need to be treated first. The next step would be to abolish any fits with intravenous benzodiazepines such as diazepam. Only if these fail to work and the patient is still in status epilepticus should other anticonvulsants, such as phenobarbital, be considered. All patients with a history of alcoholism should be given intravenous thiamine (Pabrinex) as they are likely to be lacking this vitamin from their diet. Prompt administration could prevent the development of Wernicke's encephalopathy. Propranolol can be used to treat tremor and Haloperidol can be used to treat agitation but both drugs should not be used in this patient. Haematology EMQ 1- Peripheral blood film A. Howell-Jolly bodies B. Auer rods C. Spherocytes D. Smear cells E. Heinz bodies F. Sickle cells G. Red cell agglutination H. Atypical lymphocytes I. Hypersegmented neutrophils J. Red cell fragments Choose the most relevant and likely blood film abnormality in the following scenarios.

Page 45: 80 Neuro Resp Gastro Haem Questions

www.AceMedicine.com

1. A 24 year old male with tiredness. His full blood count reveals an anaemia (Hb 9.5 g/dL) and severe thrombocytopenia (platelet count 5x 10^9/L). He reports fevers and headaches in the last 2 days and passing dark brown urine. His creatinine is raised at 150 umol/L. While waiting in the ED, he has a seizure. J- This patient has features that suggest TTP (thrombotic thrombocytopenic purpura), which is a haematological emergency associated with a high mortality due to cardiac and CNS micro-vessel ischaemia resulting from fibrin-rich clots. It is characterized by a pentad - MAHA (micro-angiopathic haemolytic anaemia, characterized by red cell fragmentation and polychromasia), thrombocytopenia, renal impairment, fever, and neurological symptoms. Note that the pentad is not always present and is not necessary for the diagnosis. The differential includes other causes of MAHA, including malignancy, sepsis (meningococcal sepsis needs to be excluded here and the patient empirically treated after blood cultures are taken as an LP would be contraindicated with his low platelet count), malignant hypertension and the haemolytic uraemic syndrome (HUS). Treatment of TTP is with immunosuppression and plasma exchange. Platelets should not be administered. 2. A 68 year-old female has an FBC after attending her GP for a recent episode of productive cough. The sample produces an unusually high MCV and MCHC, which corrects on warming. Atypical serology is positive for mycoplasma pneumoniae. G- The most likely cause of these laboratory abnormalities are red cell agglutination invitro at room temperature, which is caused by a cold agglutinin. This is usually an IgM antibody that cross-reacts to red cell antigens, typically anti-I, at low temperatures, typically <20 deg Celcius. The IgM antibody being a pentamer, is able to crosslink and agglutinate red cells. This antibody is most often caused by infections, including bacterial, eg Mycoplasma pneumoniae and viral e.g Herpes simplex. Mycoplasma causes a seasonal atypical pneumonia, which this patient is likely to have. The cold agglutination often has no effect on the patient, although cold agglutinins can sometimes cause haemolysis, especially if the antibody is active in the cooler peripheries or in cold weather. Cold agglutinins are also associated with lymphoproliferative disorders. 3. A man who had an emergency splenectomy after an RTA A- Howell-Jolly bodies are small blue round red cell RNA inclusions. These are usually removed by the spleen as the cell matures, such that a hypofunctioning or absent spleen leads to persistence of these nuclear remnants. Other features of hyposplenism include a raised platelet count, red cell shape abnormalities and a lymphocytosis. The patient should receive penicillin V prophylaxis and be immunized against encapsulated organisms (e.g Neisseria, Pneumococcus and Haemophilus). 4. A 12 year old Caucasian boy with chronic anaemia and a history of neonatal jaundice and who has recently developed episodes of RUQ pain. His sister required a splenectomy at the age of 10.

Page 46: 80 Neuro Resp Gastro Haem Questions

www.AceMedicine.com

C -The history suggests an inherited anaemia given the long history, neonatal jaundice and the fact that his sister required a splenectomy. The common causes that present with neonatal jaundice include HS and G6PD deficiency. The latter, although associated with Heinz bodies, does not usually require splenectomy and is rarer in the Caucasian population, being more common in the Mediterranean and African population. The diagnosis is therefore likely to be HS, in which numerous spherocytes are seen, which appear as hyperdense round red cells without a clear central pale area. He has probably developed pigment gallstones from the increased red cell turnover causing his abdominal pain. One would expect a raised unconjugated bilirubin. 5. An 85 year old man presents with lethargy. He has the following results: Hb 5.4 MCV 124 Plt 100 WBC 3.4 Neutrophils 1.3. His vitamin B12 level is low. I -This man has megaloblastic anaemia with a pancytopenia secondary to B12 deficiency. Hypersegmented neutrophils are a common feature in B12 or folate deficiency, arising due to abnormal nuclear maturation due to defective DNA synthesis. He should be started on IM hydroxocobalmin replacement. A cause for his B12 deficiency should be sought, the commonest of which will be dietary or malabsorption secondary to pernicious anaemia or small bowel pathology. PA can be diagnosed using anti-intrinsic factor antibodies (specific but not sensitive) and gastric anti-parietal antibodies (sensitive but not specific). A Schilling’s test could also be performed although this is less often done nowadays. Most patients remain on lifelong three monthly B12 replacement. Haematology EMQ 2-Haematological malignancy A. Myeloma B. Acute promyelocytic leukaemia C. Burkitt lymphoma D. Follicular lymphoma E. Chronic lymphocytic leukaemia F. Chronic myeloid leukaemia G. Acute lymphoblastic leukaemia H. Waldenstrom’s macroglobulinaemia I. Monoclonal gammopathy of unknown significance J. Acute myeloid leukaemia H. Myelodysplasia 1. A 70 year old presents with lymphadenopathy and splenomegaly and investigations reveal an IgM paraprotein of 40g/L. Hb 10g/L. Creatinine and calcium normal. Skeletal survey shows no bone lesions.

Page 47: 80 Neuro Resp Gastro Haem Questions

www.AceMedicine.com

H A paraprotein is most often seen in myeloma, although a few other lymphoproliferative disorders can be associated with this. True IgM myeloma is very rare (frequency: IgG>IgA>IgD>IgM). More likely is that this patient has a type of lymphoma (lymphoplasmacytic lymphoma) producing excess IgM. Collectively the syndrome is called Waldenstrom’s macroglobulinaemia, which usually also presents with bone marrow infiltration, splenomegaly and sometimes enlarged lymph nodes, but not lytic bone lesions or hypercalcaemia. The IgM can cause hyperviscosity due to its size. 2. A 45 year old man presents with back pain, night sweats, weight loss and a 10cm retroperitoneal mass. He has a markedly raised LDH and his HIV result returns as positive. Histology demonstrates a ‘starry sky’ pattern and most of the cells are in mitosis. C. These features suggest Burkitt lymphoma, a highly aggressive and rapidly proliferating mature B cell tumour, hence the high LDH. It has a strong association with HIV, which probably contributes to oncogenic changes in the cell by causing increased B cell proliferation/expansion. It can present with abdominal masses or rapidly enlarging lymphadenopathy. B symptoms are often present. 3. A 76 year old man presents with bruising and gingival bleeding and general lethargy. His clotting profile is normal. A blood film shows pancytopenia with immature hypogranular blasts, which stain positively for myeloperoxidase. Occasional Auer rods are seen. Cytogenetics shows t(8;21) J. This patient has acute leukaemia with t(8;21) translocation. The presence of Auer rods is pathognomic of acute myeloid leukaemia and myeloperoxidase is expressed by myeloblasts. Acute promyelocytic leukaemia is a subtype of acute myeloid leukaemia, most often presenting with a coagulopathy/DIC picture with prominent bruising/bleeding. The blasts in APML are often hypergranular and it is associated with the t(15;17) translocation causing the PML-RARA fusion protein. 4. An elderly man presents with Coomb’s test positive haemolytic anaemia. He has a lymphocytosis of 20 X 109/L. A blood film shows small mature lymphocytes, smear cells and spherocytes. E. The picture is consistent with a lymphproliferative disorder and the most likely diagnosis is CLL. CLL can be associated in about a 10% of cases with a Coomb’s test positive haemolytic anaemia (hence the spherocytes). Other lymphoproliferative disorders can cause peripheral blood lymphocytosis (e.g mantle, follicular) but these are much less common than CLL, which is by far the commonest LPD. CLL cells are fragile and easily ‘smudged’ or ‘smeared’ during film preparation. Diagnosis can be confirmed by doing peripheral blood immuno-phenotyping which shows up characteristic pattern of surface markers on the CLL cells. The bone marrow will also invariably be infiltrated with these cells. Lymphadenopathy and splenomegaly may also be present.

Page 48: 80 Neuro Resp Gastro Haem Questions

www.AceMedicine.com

5. A 75 year old who had chemotherapy for breast cancer 5 years ago has pancytopenia and macrocytosis. Bone marrow examination shows abnormal maturation in all 3 lineages. B12 and folate are normal. H The differential for a macrocytosis with pancytopenia includes B12/folate deficiency (normal here), myelodysplasia or aplastic anaemia (the marrow would be hypocellular). This elderly patient has been exposed to cytotoxic drugs 5 years ago and we are told the marrow precursors shows abnormal maturation (termed ‘dysplasia’). The diagnosis is most likely myelodysplasia. Cytogenetics, the percentage of immature blasts and the number of cytopenias are important prognostic factors. Transformation to leukaemia is the main concern here and carries a very poor prognosis. Treatment at this stage will be supportive only with blood/platelet trasfusions as needed. Haematology EMQ 3-Treatment of haemoglobinopathies Please select the most appropriate management option A. Iron chelation B. Top-up transfusion C. Exchange transfusion D. Hydroxyurea E. Hydration, oxygen and analgesia F. Admit and IV cephalosporin G. Anticoagulation H. Bone marrow transplantation I. HDU/ITU management with trial of non-invasive ventilation J. Iron replacement therapy K. No treatment required 1. A 10 year old child with sickle cell disease presents with right sided hemiparesis. A CT head shows an evolving infarct. Hb 9.0 g/dl. C. Strokes are a major cause of morbidity in young children with sickle cell disease. One should reduce the % Hb S as much as possible (ideally to <20%) with an exchange transfusion, which usually replaces most of the patient’s own sickle Hb containing red cells with Hb A containing adult red cells through a process of serial dilution with donor blood, by taking out 1 unit and replacing with another etc, whilst maintaining euvolaemia. If top-up transfusion is used, one would make the patient hyperviscous and risk further strokes. Patients who have had strokes are usually treated with chronic regular transfusions every month or so to suppress their marrow from making the abnormal HbS containing red cells. Iron overload and red cell alloimmunisation are problems that need to be managed in these patients in the longer term.

Page 49: 80 Neuro Resp Gastro Haem Questions

www.AceMedicine.com

2. Four hour acute painful bony crisis in a 23 year old with frequent sickle crises. E In the immediate setting one would manage this patient with adequate oral hydration unless the patient cannot drink (IV in that case), analgesia (patient should have a protocol, but morphine s/c or orally is usually needed; pethidine should not be used), oxygen if saturations are <95%, and searching for and treatment of infective causes if present or if patient has a fever. Incentive spirometry is useful and encourages the small airways to be kept patent by getting the patient to blow against resistance. The patient should be monitored closely for adequate pain relief and any respiratory deterioration. In the longer term, this patient may benefit from hydroxyurea. 3. Beta thalassaemia major patient who has been regularly transfused for 2 years. Otherwise well. Hb 10.5 MCV 82. Ferritin 2000. LFTs normal. A. This patient clearly has iron overload and chelation needs to be started. One should assess for complications and assess for cardiac and liver iron loading, do a glucose tolerance test and assess their annual blood transfusion requirement. Desferioxamine is the most often used drug, but requires subcutaneous infusions or intravenous infusions via central line lasting several hours a day for 5 days a week at least, causing compliance issues. A new oral chelator has been favourably compared to it and is called Exjade (Deferasirox) and is increasingly being used. 4. A 20 year old sickle cell patient is unwell and febrile with a 3-day history of cough with greenish sputum. Despite initial management with analgesia and hydration, oxygen saturations are falling <90%, she is becoming drowsy and has persistent chest pains. I. Unlike the patient in question 2, this patient is deteriorating and could be developing an acute chest syndrome. He should have a chest X-ray urgently and be considered for ventilation - non-invasive ventilation has been shown to be effective as first-line, although some patients need intubation and mechanical ventilation. One would also consider doing an urgent exchange transfusion, although the priority here would be to manage her worsening hypoxia and drowsiness. 5. A 32 year old female is 30 weeks pregnant with Hb 9.5. MCV 70. Ferritin is <10. She has a positive sickle screening test. HPLC shows 40% HbS and 60% HbA. J The ratio of Hb S and Hb A suggest this patient has sickle trait (assuming she has not been transfused, which might alter the HbS: HbA ratio). He low ferritin and microcytic anaemia suggest iron deficiency, which is not uncommon at this stage of pregnancy. She needs iron replacement. Hopefully, her partner has been tested for sickle trait and Beta thalassaemia trait as part of the National Haemoglobinopathy Antenatal screening. Both of these could lead to their newborn being affected by a sickling disrder. The sickle screening test does not differentiate between a trait and a disease state. High performance liquid chromatography or a gel electrophoresis, that separate out the different haemoglobins and allow quantitation, are more helpful than the sickle screening test,

Page 50: 80 Neuro Resp Gastro Haem Questions

www.AceMedicine.com

although the latter can be done relatively quickly in emergencies, eg pre-operative in emergency settings. Haematology MCQ- Aplastic anaemia secondary to drug therapy Which of the following have recognized associations with bone marrow failure? A. Chloramphenicol B. Sulfasalazine C. Azathioprine D. Carbamazepine E. Penicillin All of the drugs listed can cause bone marrow failure. The most important thing is to consider medications as a cause of this problem. The list of drugs that can cause marrow aplasia is long. Some are predictable and others are idiosyncratic. The commonest ones include: Diuretics/antihypertensives: Phenylbutazone; Furosemide; Nifedipine Antibiotics: Chloramphenicol; Sulfonamides Penicillin Antidiabetics: Sulfonylureas; Anti-epileptics: Carbamazepine; Valproic acid; Phenytoin Anti-platelets: Ticlopidine; Clopidogrel Allopurinol Immunosuppressives/DMARDS- Gold, azathioprine Haematology EMQ 4- Complications of blood transfusions A. Acute haemolytic transfusion reaction B. Delayed haemolytic transfusion reaction C. Bacterial contamination D. Transfusion related acute lung injury (TRALI) E. Viral hepatitis F. Anaphylactic reaction G. Febrile non-haemolytic reaction H. Transfusion associated cardiac overload I. Iron overload J. Post-transfusion purpura 1. 5 minutes into a platelet transfusion, the patient becomes acutely unwell with hypotension, high fevers and rigors.

Page 51: 80 Neuro Resp Gastro Haem Questions

www.AceMedicine.com

C. There are usually about 3-4 cases per year of platelet transfusion associated bacterial sepsis and these are often fatal. Platelets are stored at 22 degrees and if contaminated at collection from the donor allow the proliferation of bacteria. Staphylococcus, pseudomonas and streptococci are the most frequent organisms as they are skin commensals. This is currently the single most important cause of transfusion associated mortality every year in the UK. The differential should include anaphylaxis, but high fever is not usually a feature of this. 2. Seven days after a blood transfusion a patient with MDS presents with worsening jaundice and anaemia. Investigations show a positive direct Coomb’s test, unconjugated hyperbilirubinaemia and a raised LDH. B. This is a typical delayed HTR. Antibodies to non-ABO antigens present on the transfused cells that are not present in the recipient can form especially in patients who have been regularly/previously transfused and parous females previously exposed to paternal red cell antigens absent in the mother. To avoid this, the antibody ‘screen’ is an essential part of pre-transfusion testing. However, antibodies can sometimes be missed in the screen as they are at low levels or through error. These antibodies subsequently increase in response to exposure the cognate antigen and cause lysis (mainly extravascular) of donor red cells carrying the target antigen. The DCT is usually positive. Unlike acute HTR, this takes a few days to develop, but can be associated with a rapid fall in haemoglobin and features of haemolysis/jaundice. A transfusion may not be needed but if so, group specific blood negative for the antigen concerned is administered. Patients can form antibodies to several antigens and in these cases, availability of suitable blood may be difficult. 3. A patient on HDU is given 3 units of FFP to correct a coagulopathy for a procedure. 2 hours later, he develops a fever, cough and hypoxia. An echocardiogram is normal. H. TRALI is a rare, but serious complication of transfusion of plasma containing products e.g platelet and FFP. It is caused by antibodies activating neutrophils in the recipient and leading to a primarily lung-based inflammation with infiltrates that can indistinguishable to pulmonary oedema. Transfusion associated cardiac overload is the main differential and is far more common, but the normal echo here suggests TRALI to be more likely. On can investigate the donor for the neutrophil antibodies in question. Ventilation is often needed. 4. A patient complains of back pain and develops rigors and hypotension within 5 minutes of a blood transfusion having commenced. He is blood group O when his group is re-checked and the bag states ‘group A’. A. There is a major incompatibility in ABO here and this has caused an acute HTR, which is a life threatening emergency. One should immediately stop the transfusion and resuscitate the patient with IV fluids and maintain blood pressure and urine output. This can be fatal. The patient should be managed on HDU and treatment is primarily supportive. This incident will need to be reported centrally to a national haemo-vigilance programme and root-cause analysis of this error performed.

Page 52: 80 Neuro Resp Gastro Haem Questions

www.AceMedicine.com

This could have occurred at several stages of the transfusion process, though here the wrong group/crossmatch sample was sent and the patient was grouped as A instead of O as a result. Previous records were not available for the error to be highlighted pre-issue of the blood unfortunately. 5. An 80 year old has a macrocytic anaemia with a low B12 level. She is admitted with tiredness which has been gradually developing for the last month. Her Hb is 5.8 g/dl. She is prescribed 3 units of blood. After the 2nd unit she becomes acutely breathless and a chest X-ray reveals bilateral perihilar infiltrates. H This elderly patient in fact should not have received any blood. Her anaemia is chronic and she has haemodynamically adjusted to this. These patients respond well and fairly quickly to vitamin replacement. A large blood transfusion and probably some element of cardiac impairment given her age has led to fluid overload. She should be diuresed and treated with oxygen. An echocardiogram may help establish cardiac impairment although this is not a requirement. Haematology EMQ 5- White cell disorders Select the most likely diagnosis from the following A. Mantle cell lymphoma B. ALL C. AML D. Myelofibrosis E. Myeloma F. Myelodysplasia G. Diffuse large B cell lymphoma H. Follicular lymphoma I. Chronic myeloid leukaemia J. Chronic lymphocytic leukaemia 1. Male patient with lethargy, splenomegaly and the following blood count: WBC 60 (Neutrophils 55% myelocytes 25% Lymphocytes 2% Eosinophils 5% Basophils 1% Monocytes 1% Blasts 1%); platelets 600; Hb 11.5. BCR-ABL translocation is detected by FISH analysis on peripheral blood. I. This patient has CML with a marked leucocytosis with a myelocyte and neutrophil peak. There is also an eosinophilia, excess basophils and platelets as well as the occasional blast. She has a myeloproliferative disorder, with CML being the most likely diagnosis. Philadelphia chromosome analysis or BCR-ABL detection by FISH confirms the diagnosis.

Page 53: 80 Neuro Resp Gastro Haem Questions

www.AceMedicine.com

2. Male patient with lethargy and 16cm splenomegaly. He has a leucoerythroblastic blood film. Bone marrow - unable to aspirate (dry tap). Marked increased in reticulin on trephine. D. These features suggest marrow fibrosis and the markedly enlarged spleen suggests myelofibrosis. Other causes of marrow fibrosis/leucoerythroblastosis includes; infiltration with malignancy (usually localized) and TB. Other causes of massive splenomegaly are visceral leishmaniasis and Gaucher’s disease (storage pool disorder). 3. Male patient with back pain and normocytic anaemia Hb 8 g/dl, WBC/platelet count normal. Creatinine 200 mmol//. MRI spine shows a pathological fracture of T7. Serum protein electrophoresis: negative. Bence Jones protein (BJP) detected in urine. E. A proportion of myelomas (about 10%) do not produce heavy chains, such that intact monoclonal immunoglobulin (ie. a paraprotein) is not present. However, in the majority of these cases produce light chains (kappa or lambda) with resultant cast nephropathy and positive BJP in the urine. That is why a BJP should always be part of the myeloma screen. A very small proportion of cases (<5%) do not even produce measurable amounts of BJP (oligo or non-secretory myelomas)- these are more difficult to diagnose. Serum free light chain assays are helpful as they are far more sensitive that the urine free light chain test (BJP). 4. Patient with HIV with a mediastinal and retroperitoneal mass, weight loss, sweats and fevers. G This patient could have lymphoma or TB or another malignancy. The commonest lymphoma in HIV is diffuse large B cell lymphoma and this is therefore the most likely. A biopsy of the mass will be needed to confirm the diagnosis. 5. A 3 year old boy with bruising and irritability. He has a white cell count of 100 x 10^9/L, Hb 5.7 g/L platelets 20 ml and hepato-splenomegaly. A blood film is shows a monomorphic population of small blasts with no granulation and scanty cytoplasm. Myeloid surface markers are not detected. B. This child has ALL with marrow failure as a result of infiltration with lymphoblasts. ALL is the commonest tumour is this age group. The presentation is typical. Immunophenotyping is needed to confirm that this is not AML, although the morphology is highly suspicious of ALL. Auer rods do not occur in ALL.

Page 54: 80 Neuro Resp Gastro Haem Questions

www.AceMedicine.com

Haematology EMQ 6- Macrocytosis

A. Hypothyroidism B. Autoimmune haemolytic anaemia C. Alcohol D. B12 deficiency E. Folate deficiency F. Myelodysplasia G. Aplastic anaemia H. Drug-induced macrocytosis I. HIV

1. A 32 year old man is brought in collapsed to A&E with multiple bruises, smelling of alcohol and extremely unkempt. He is pancytopenic with a raised INR and deranged LFTs. MCV is 125fL. There are hypersegmented neutrophils on the film. Serum B12 is normal. E. Alcoholics and the homeless tend to be malnourished and folate deficiency is common due to poor intake of greens and vegetables rich in folic acid. Note that alcohol and liver disease themselves can cause a macrocytosis and low counts, but rarely cause an MCV as markedly raised as this; the hypersegmented neutrophils also point to megaloblastic anaemia. B12 deficiency has been excluded although combined nutritional deficiencies are not unusual. He should be given Pabrinex and folate. 2. A 62 year old man presents with angina. He is anaemic and jaundiced with; Hb 6.2 g/dl, MCV 106 fl, reticulocytes 5%. A blood film shows polychromasia, spherocytes and nucleated red cells. B. The features here are of haemolysis. Although we are not given the result of a direct Coomb’s test, the most likely cause is of this picture is autoimmune haemolysis. Spherocytes are the product of red cells that have bits of membrane removed by the spleen as a result of antibody coating. One should look for underlying lymphoproliferative and immune disorders. The younger red cells (reticulocytes) are increased in haemolytic states and tend to be larger in size, hence making the MCV high. They also tend to be bluer on film stains compared to more mature red cells, causing polychromasia. 3. An 80 year old man has been noted to be becoming more anaemic over the last year with a progressive macrocytic picture. The MCV currently 102 fl. Reticulocytes are normal. B12 and folate are normal. DAT is negative. F. Given this man’s age and the important negatives we are provided, which exclude haemolysis and megaoblastosis, myelodysplasia is the most likely cause. One should check his thyroid function and get a detailed drug history.

Page 55: 80 Neuro Resp Gastro Haem Questions

www.AceMedicine.com

4. A 40 year old female presents with weight gain, hair loss and lethargy. Deep tendon reflexes are reduced. Her MCV is 102fL and she has a mild anaemia. A. These are features of severe hypothyroidism. One should look for an immune cause e.g Hashimoto’s. A mild macrocytosis can be found as part of severe hypothryroid states, although relatively rare and non-specific. The cause is not clear. 5. A 33 year old man has recently started AZT(azidothymidine). His retroviral titre is undetectable, but he has become more anaemic and thrombocytopenic with a macrocytosis. H. Drugs such as AZT is a well-recognized cause of macrocytosis and cytopenias with reversible megaloblastic change in the marrow. Other agents include cytotoxics, including hydroxyurea. HIV itself can cause cytopenias and dysplasia though this patient has no detectable viral load. Haematology SBA A 46 year old man is admitted with a 2 month history of malaise, left upper quadrant discomfort and weight loss. His peripheral blood film shows noticeable teardrop poikylocytes, few myelocytes and occasional blasts and nucleated red cells. The most likely abnormality present on further investigation is? A. Auer rods on bone marrow aspirate B. Serum paraprotein on protein electrophoresis C. Philadelphia chromosome on cytogenetic analysis D. Generalised lymphadenopathy on CT E. Extensive fibrosis on bone marrow trephine reticulin stain E. Extensive fibrosis on bone marrow trephine reticulin stain These features suggest leucoerythroblastic change, a feature of marrow infiltration (primary myelofibrosis, cancer, granulomas, myeloma, storage pool disease). The teardrop poikylocytes are quite typical, however of myelofibrosis. Reticulin and collagen fibrosis is likely to be seen on histology of the bone marrow, confirming a diagnosis of myelofibrosis. The other features would be absent.

Page 56: 80 Neuro Resp Gastro Haem Questions

www.AceMedicine.com

Haematology MCQ Which of the following are features of untreated iron deficiency on a blood film? A. Target cells B. Microcytosis C. Pencil cells D. Polychromasia E. Hypersegmented neutrophils A,B,C Target cells are red cells with extra membrane relative to haemoglobin/cytoplasm, such that the membrane has extra folds. When seen on a film, they appear like targets. They are not specific for iron deficiency and can also be seen in haemoglobinopathy states, sickle cell disease, liver disease, splenectomy/hyposplenism. Polychromasia can be seen when iron therapy is started and is associated with a reticulocyte response. Hypochromia and anisocytosis are also invariably seen. Hypersegmented neutrophils are a feature of megaloblastic aneamia. Haematology SBA A 65 year old female is noted to have the following full blood count- Hb 14 g/dl (NR 11-15), Plt 800 x 10^9/L (NR 150-450), WBC 20 x 10^9/L (NR 4-9) Differential: neutrophils 50% eosinophils 30% basophils 5% lymphocytes 10% monocytes 5%. She has 3 cm splenomegaly. She has been having transient clouding of vision in her right eye. Which one of the following best explains the above clinical picture and differential count? A. Eczema B. Myeloproliferative disorder C. Lymphoma D. TB E. Vasculitis B. The differential shows a high WBC with an increase in myeloid lineage elements (neutrophils, eosinophils, basophils). There is also a significant thrombocytosis. Although this picture could be reactive to infection or inflammation, splenomegaly would be unusual. The history also is concerning for amaurosis fugax, which may be related to the high platelet count- this is associated with increased stroke risk. A myeloproliferative disorder is most likely. She needs to receive aspirin and after the diagnosis is confirmed with a bone marrow, go on to cytoreductive therapy with hydroxyurea to normalize her counts. Other conventional cardiovascular risk factors should be sought and treated these and give lifestyle (including smoking) advice.

Page 57: 80 Neuro Resp Gastro Haem Questions

www.AceMedicine.com

Haematology MCQ A 76 year old man presents with weight loss of 4kg and sweats. His full blood count shows an eosinophilia. Possible causes for this picture are: A. Drugs B. Myeloproliferative disorder C. Lymphoma D. Strongyloidosis E. Carcinoma A,B,C,D,E, All can cause an eosinophilia. Eosinophilias are most often reactive, caused by infections, particularly parasites including worms, allergic states such as asthma and eczema, drugs which may be otherwise asymptomatic and vasculitis (such as Churg Strauss and Wegener’s). Lymphomas and cancers sometimes produce cytokines including IL-5 that drive eosinophil proliferation. Primary causes where there is no extrinsic stimulus to drve eosinophil proliferation include myeloprpliferative disorders. Persistently high eosinophil counts can cause tissue infiltration and damage from their enzyme containing granules. Haematology SBA An 18 year old women has menorrhagia and iron deficiency anaemia. She also has a long history of easy bruising and epistaxis since childhood requiring cautery. She bled excessively after tonsillectomy at age 13. Her mother bruises easily. A coagulation screen shows are normal PT and APTT. Her platelet count is normal. What is the most likely diagnosis? A. Haemophilia B. Von Willebrand disease C. Connective tissue disorder D. Inherited platelet function defect E. No significant bleeding disorder B. Von Willebrand disease There is a possible familial tendency to bleed and bruise easily. The nature of bleeding is mucosal, making a platelet problem and von-Willebrand disease more likely than a factor defect/haemophilia. Von willebrand disease, especially type 1, is associated with mild-moderate low levels of VWF and is

Page 58: 80 Neuro Resp Gastro Haem Questions

www.AceMedicine.com

by far the most common bleeding disorder for which the history is typical. Inherited platelet function defect needs to be excluded if von willebrand studies are normal. Treatment with antifibrinolytics eg Tranexamic acid is helpful in mild cases such as this one around the time of periods. One may also suppress the patient’s menses with oral contraceptive pills if very heavy. Haematology MCQ Which of the following are often associated with soft tissue haematomas and haemarthroses if untreated? A. Severe Haemophilia A B. Immune thrombocytopenia C. Thrombotic thrombocytopenic purpura D. Type 1 von Willebrand disease E. Type 3 von Willebrand disease A, E Deep soft tissue and joint bleeds are not a feature of platelet problems (ITP) and von willebrand disease, with the exception of type 3 VWD, where the levels of VWD are extremely low/undetectable. As VWF carries and stabilizes Factor VIII, this effectively behaves like severe Haemophilia A or factor VIII deficiency, which present with haematomas and joint bleeds which can be spontaneous or on minimal injury. Factor replacement is required. TTP is paradoxically associated with thrombosis rather than bleeding. Haematology MCQ A 43 year old female present with epistaxis, bruising and menorrhagia for 1 week. She is febrile and appears unwell. She is diagnosed with acute promyelocytic leukaemia. Which of the following are the most important aspects of her initial management? A. Broad-spectrum antibiotics B. Platelet transfusion to keep platelets > 50 x 10 9/L C. Cryoprecipitate to keep fibrinogen> 1g/dL D. All-trans retinoic acid E. Blood transfusion to keep Hb >8 g/dL B,C,D,E

Page 59: 80 Neuro Resp Gastro Haem Questions

www.AceMedicine.com

Managing the coagulopathy (DIC and hyperfibrinolysis) is essential as early mortality results from bleeding primarily. This patient is not septic, so no antibiotics are needed until she spikes a temperature. ATRA has been shown to cause differentiation an elimination of the promyelocytes that cause the problem. Haematology SBA A 4 year old boy presents with pallor, petechiae and a painful limp. He has lymphadenopathy and a chest X-ray shows a widened mediastinum, reported as a possible mass. The most likely diagnosis is ? A Hodgkin’s disease B. Acute lymphoblastic leukaemia (ALL) C. Acute myeloid leukaemia (AML) D. Mediastinal B cell lymphoma E. TB B. ALL ALL is the most likely cancer at this age. Children often present with leg/bone pain and a limp with non-specific lethargy, poor feeding and irritability. Causes of a mediastinal mass include lymphoma, TB, retrosternal goitre, lung or other malignancy (including teratoma in a child) and thymoma. Thymomas and goitres tend are in the anterior mediastinum. Other general causes of mediastinal widening include aortic dissection/aneurysm (unlikely in a child). Haematology SBA Which one of these is the most sensitive test in making a diagnosis of lymphoma? A. CT-PET scan B. Bone marrow aspirate and trephine (BMAT) C. LDH D. Excision biopsy of abnormal/enlarged lymph node E. Fine needle aspiration of abnormal/enlarged lymph node D. Excision biopsy of abnormal/enlarged lymph node Differentiate between staging and diagnosis. For the latter one needs tissue for histology and an excision biopsy is most useful. FNAs should not be performed, although they can be useful in other

Page 60: 80 Neuro Resp Gastro Haem Questions

www.AceMedicine.com

tumours e.g thyroid. The BMAT and LDH as well as CT-PET scan are staging tests that tell us the extent of disease. Haematology MCQ A 45 year old man has a platelet count of 2000 X 10^9/L and is diagnosed with essential thrombocytosis. Which of the following are recognized complications of this disorder? A. Bleeding B. DVT C. Stroke D. Progression to myelofibrosis E. Transformation to acute lymphoblastic leukaemia A, B, C D Bleeding due to an acquired von willebrand disease, whereby the VWF adheres to the surface of the increased platelets, with reduced circulating levels, is well recognized. Thrombotic complications are common. With time (usually years) progression to myelofibrosis and AML, but not ALL, can occur. Haematology SBA A 50 year old man is found to have a haematocrit of 0.56 % (NR 0.45-0.52). He is a heavy smoker and has COPD and suffers from hypertension controlled with diuretics. He has been admitted with an infective exacerbation of COPD. He has a raised white cell count with a neutrophilia. JAK-2 mutation analysis is negative. The least likely cause for this man’s polycythaemia is ? A. dehydration B. lung disease C. diurectics D. polycythaemia rubra vera E. smoking D. First differentiate between true and relative polycythaemia. True polycythaemia (raised red cell mass) can be primary or secondary. Dehydration and diuretics (A and C) can cause a relative

Page 61: 80 Neuro Resp Gastro Haem Questions

www.AceMedicine.com

polycythaemia, where there is relatively low plasma volume: red cell mass ratio. Red cell mass (normal) and plasma volume (reduced) studies is helpful to show a relative polycythaemia. This man also has factors for secondary polycythaemia, including COPD and smoking. Other causes are obstructive sleep apnoea, cardiac shunts (eg congenital heart disease) and more rarely EPO-producing tumours (renal, haemangioblastimas, fibroids) and drugs including exogenous androgens (hence higher Hb in males v/s females!). Nowadays the JAK-2 mutation analysis is central to evaluating primary polycythaemia, as it has been shown to be present in >95% of cases of primary polycythaemia rubra vera. This is therefore the least likely diagnosis here. Raised EPO levels can sometimes help support secondary polycythaemia. Haematology SBA An elderly gentleman complaining of tiredness is found to have a sensory neuropathy and proteinuria of 2.8 g/24h. Urine analysis shows lamda Bence Jones protein. Amyloidosis is suspected. The most appropriate test to confirm the diagnosis is ? A. Abdominal ultrasound B. Bone marrow aspirate C. Skeletal survey D. Echocardiogram E. Renal biopsy E. Amyloidosis is a multisystemic disorder. The presence of a paraprotein and/or BJP is a clue to a possible diagnosis of AL amyloid, which is most commonly associated with plasma cell dyscrasias (eg myeloma, MGUS) Neuropathy secondary to amyloid deposition in the nerve sheath and blood vessels supplying the nerve is one feature, although this can also result from other factors that need evaluation here (eg drugs, alcohol, diabetes). This patient also has near-nephrotic range proteinuria, suggesting renal involvement. The most likely place to pick up amyloid deposition here is a renal biopsy, with demonstration of red/green birefringent amorphous deposits on Congo red staining, when viewed with polarized light. The other tests mentioned are also relevant to assess the cause and extent of the disease but will not be diagnostic. Nowadays a radiolabelled SAP (serum amyloid P) scan is useful at detecting the degree and extent of tissue deposition of amyloid, particularly in lever and spleen.

Page 62: 80 Neuro Resp Gastro Haem Questions

www.AceMedicine.com

Haematology MCQ Which if the following are recognized manifestations of amyloidosis? A. Hepatosplenomegaly B. Cardiomyopathy C. Postural hypotension D. Renal failure with small kidneys on ultrasound E. Constipation A,B,C,E All except D can occur in this multi-systemic disorder. C and E are due to involvement of the autonomic nerves supplying the blood vessels and the bowels. Typically the kidneys are enlarged due to amyloid deposisition. Cardiac complications are the commonest cause of death. Haematology MCQ In which of the following situations would the ESR typically be raised? A. Severe anaemia B. Pregnancy C. Polycythaemia D. Metastatic cancer E. Polymyalgia rheumatica A,B,C,E The ESR is a measure of how quickly red cells ‘fall’ when suspended in a vertical column, and is expressed in mm/h. The rate of fall is affected by interactions between the red cells themselves and the plasma proteins in which they are suspended. Factors that allow the red cells to fall more easily include the formation of rouleaux (stacking together of red cells into column like structures) as a result of changes in red cell-red cell interactions- usually red cells have like –ve charge and repel each other, but plasma proteins eg acute phase reactants and immunoglobulin’s can sometimes overcome this allowing such stacking to occur. This is seen in severe anaemia, cancer, pregnancy and inflammatory conditions like PMR. In polycythaemia, the red cells ‘bounce’ off each other as they fall in a more crowded field and the ESR is low.

Page 63: 80 Neuro Resp Gastro Haem Questions

www.AceMedicine.com

Haematology SBA A 50 year old man is referred with headaches and blurred vision. He has a normocytic anaemia with Hb 10g/dL. Right temporal artery pulse is not palpable. His ESR is 120 mm/s. The most important aspect of his management is ? A. Start corticosteroids B. Refer to ophthalmologist C. Check serum electrophoresis D. Skeletal survey E. Carotid artery Doppler A. Start corticosteroids This is an emergency- the patient has symptoms and signs of temporal arteritis (also known as PMR, or giant cell arteritis) and needs urgent treatment to reduce the risk of a stroke. One should not delay starting steroids, although it would be important to get a rheumatology and ophthalmology opinion too. Haematology SBA Which one of the following paraproteins is most likely to cause hyperviscosity symptoms? A. IgA B. IgD C. IgE D. IgG E. IgM E.IgM IgM because of the pentameric structure of the IgM molecule (very large). This is not always related to the level of the paraprotein. IgG and IgA can also cause hyperviscosity.

Page 64: 80 Neuro Resp Gastro Haem Questions

www.AceMedicine.com

Haematology MCQ A 60 year old has been diagnosed with a paraprotein. Recognized features of hyperviscosity are? A. Epistaxis B. Blurred vision C. Joint pains D. Headache E. Limb ischaemia A, B, D Hyperviscosity causes small vessel problems as the blood flow becomes sluggish. There may be capillary congestion as a result and bleeding, retinal symptoms and headaches are common. Limb ischaemia is a large vessel problem and does not occur. Joint pains are not a feature. Haematology SBA A 20 year old has develops a deep vein thrombosis. She was on the combined oral contraceptive pill. What is the next step in management? A. Urgent thrombophilia test before starting anticoagulation B. Start unfractionated heparin C. Start warfarin D. Start low molecular weight heparin E. Start aspirin D. Start low molecular weight heparin The combined pill should be stopped (progesterone only pill not associated with increased risk). She should be started on low molecular weight heparin in the first instance and counselled about warfarin. She should have a pregnancy test before starting warfarin and be warned about the risk of becoming pregnant whilst on warfarin in terms of fetal toxicity (highest between 6-18 weeks), drug interactions and bleeding risk. She can be managed in the outpatient setting. Aspirin is not effective in treating and preventing DVT. Unfractionated heparin would require an infusion in hospital. There is no reason to do an urgent thrombophilia screen. She was on the oestrogen containing OCP, which is a risk factor for VTE. If there is a family history of VTE one should consider doing a thrombophilia screen, around 1 months after completing 6 months of anticoagulation, when factors that might interfere with the assays (acute clot, anticoagulants) would not be present.

Page 65: 80 Neuro Resp Gastro Haem Questions

www.AceMedicine.com

Haematology MCQ Which of the following are associated with the anti-phospholipid syndrome? A. Recurrent miscarriages B. Ischaemic stroke C. Venous thrombosis D. Bleeding tendency E. Lupus anticoagulant A,B,C,E This syndrome presents with both arterial and venous thrombotic problems and/or adverse pregnancy outcomes (miscarriages, pre-eclampsia, fetal death). It can occur in isolation, but is associated with SLE. Laboratory features include demonstration of a lupus anticoagulant activity (prolongation of a phospholipid dependent test in vitro, due to the anti-phospholipid action of the LA). Not all anti-phospholipid antibodies do so and one should also check for the presence of anti-phospholipid antibodies using an ELISA test. These are typically directed against cardiolipin and beta-2-glycoprotein and can be IgG or IgM. High levels of these antibodies and lupus anticoagulants are correlated with thrombosis risk in-vivo as their effect is to activate endothelium and platelets. They don’t cause an anticoagulant/bleeding risk in vivo. Haematology MCQ Recognised side effects of cyclosporin include? Hypertension Gum hyperplasia Headaches Deranged LFTs Renal impairment A,B,C,D,E Cyclosporin is used as an immunosuppressant in rheumatological conditions, but also allogeneneic transplant recipients. It has a narrow therapeutic range and trough/peak levels should be monitored. Renal impairment and hypertension is dose/level related as are headaches. Cerebellar toxicity can also occur with high levels.

Page 66: 80 Neuro Resp Gastro Haem Questions

www.AceMedicine.com

Haematology MCQ A patient has recently been started on steroids for immune thrombocytopenia.

A. Which of the following side effects are they likely to experience in the first 2 weeks? B. Osteopenia C. Hyperglycaemia D. Sleeplessness E. Hypotension F. Cushing’s syndrome

B,C. These are frequent short term side effects of steroids. A and E occur when steroids are used for medium to long term. Avascular necrosis is also a bone related complication typically affecting children whose epiphyses have not closed yet and who are treated with high doses of steroids. Hypertension can also occur in the short term.

Editors: Dr Ian Bickle & Dr Paul Hamilton

Question Writers:

Neurology by Jon Rohrer Respiratory by Ian Bickle Gastroenterology by Dr Aruna Dias Haematology by Sajir Mohamedbhai